Consti 2 Digests (Finals) (Final Edit)

You might also like

Download as ppt, pdf, or txt
Download as ppt, pdf, or txt
You are on page 1of 124

Ynot v.

Intermediate Appellate Court


[POLICE POWER/PROCEDURAL DUE PROCESS]

Facts:
Executive Order 626-A prohibited the inter-provincial movement of carabaos and carabeef in the interest
of protecting and conserving these beasts of burden for small farmers who need them to do work. Thus, the law called
for the immediate confiscation and forfeiture of transported carabaos and carabeef for distribution to charitable
institutions.

Issues:
1. Is the law a valid exercise of police power?
2. Does it violate the due process clause?

Ruling:
1. NO. The test for the valid exercise of police power requires that there be a (a)lawful subject and
(b)lawful means. Indeed, there is a need to protect carabaos from slaughter, as they are the primary workhorses of our
country’s struggling farming industry—as such, there is lawful subject. However, EO 626-A punishes the movement,
and not the slaughter of carabaos, a prohibition whose object escapes even the Supreme Court. “We do not see how
the prohibition of inter-provincial transport of carabaos can prevent their indiscriminate slaughter, considering that they
can be killed anywhere, with no less difficulty in one province than in another.” Therefore, EO 626-A violates the
requirement for lawful means, and as such is NOT a valid exercise of police power.

2. YES. To give substance to a concept that is otherwise intentionally left vague by the courts, due
process has been given two minimum requirements for its validity: hearing and trial. In this case, there is no such
pressure of time or action calling for the immediate confiscation of carabaos, as doing otherwise would not immediately
endanger the general public. There was no reason why the violation of EO 626-A could not be resolved first in a court of
justice, where the accused will be accorded all the rights safeguarded to him by the Constitution.
Beltran v. Secretary of Health
[POLICE POWER/EQUAL PROTECTION]

Facts:
Republic Act 7719, or the National Blood Services Act of 1994, in attempting to regulate blood donations
and ensuring an adequate supply of safe and quality blood in hospitals, sought to phase out commercial blood banks,
as research has found that paid blood donors are more likely to lie about their age and medical condition than voluntary
donors, thus tainting the quality of blood donations offered by commercial establishments. Owners of such blood banks
contended that the phase-out plan is against the constitutional guarantee of equal protection, because it unduly
discriminated against them in a manner not germane to the purpose of the law.

Issues:
1. Is the act of phasing out commercial blood banks a valid exercise of police power?
2. Does the law violate the equal protection clause?

Ruling:
1. YES. In testing whether RA 7719 is a valid form of police power, it is important to note that there is
little question as to its lawful subject. Regulating the quality of donated blood affects every citizen who might need
transfusions for whatever purpose. It is the interest of the general welfare, and not just of a particular class, that it at
stake, therefore the interference of the State is required. Moreover, the means employed are reasonably necessary and
not unduly oppressive upon individuals. In the visors of police power, deprivation of property is valid when done for the
good of the totality of the populace.

2. NO. For an act to be in conformity with the equal protection clause, it must be:
1)Based on substantial distinctions
2)Germane to the purposes of the law
3)Applicable to future conditions (not just to existing conditions)
4)Must apply equally to all members of the class

On one hand, government blood donation programs, like that of the Philippine Red Cross are operating purely for
humanitarian reasons, whereas commercial blood banks are aimed towards profit, a clear and substantial distinction.
Thus the phase-out of commercial blood banks is not an assault on equal protection of the law, but rather for the
enhancement and strict adherence of professional and scientific standards of blood collection, as well as to regulate the
implementation of voluntary blood donation programs by the government; as such it is germane to the purpose of RA
7719.
PRC v. De Guzman
[Police Power]

Facts:
The PRC withheld the registration and induction of some graduates of the Fatima College of Medicine
after its investigative body, in cooperation with the National Bureau of Investigation, determined that the students have
gained early access to test questions for two of the most difficult subjects in the Physician Licensure Examination. The
students revolted and asked in a petition for mandamus to have their names registered as physicians, as is their
supposed right.

Issues:
1. Is the act of denying the students’ licenses a valid exercise of police power?

Ruling:
1. YES. While it may be true that the Supreme Court has upheld the constitutional right of every citizen to
select a profession or course of study, this right is nevertheless subject to fair, reasonable, and equitable admission and
academic requirements. Like all rights and freedoms guaranteed by the charter, exercising the right to a profession may
be regulated pursuant to the police power of the State to safeguard health, morals, peace, education, order, safety, and
general welfare. The present case clearly falls squarely within the grounds for denying a license to practice medicine as
provided under RA No. 2382, otherwise known as the Medical Act of 1959. Moreover, the act also passes the tests of
lawful subject and lawful means, prohibition being the most logical and least pervasive solution to the injustice of
admitting unqualified persons to the medical board.
MMDA v. Bel-Air Village Association
[POLICE POWER]

Facts:
On December 30, 1995, the Bel-Air Village Association received a notice from the MMDA compelling the
former to open Neptune Street, a private road within a private subdivision in the heart of the commercial district of
Makati, to public traffic, pursuant to the MMDA’s exercise of police power. Petitioners claim that being an agent of the
state, the police power is extended to their prerogative.

Issues:
1. Does the MMDA possess police power?

Ruling:
1. NO. The Supreme Court held that the MMDA does not have the authority to exercise police power on
behalf of the State. Police power is delegated from the National Legislature to some of its agents, including Local
Government Units. Republic Act 7924 declared Metro Manila as a special development and administrative region and
created the MMDA as a special development authority to administer metro-wide basic services affecting the region,
including formulating, regulating, and coordinating the implementation of medium and long term developmental plans for
the metro. However, no syllable in this law expressly confers police power, let alone legislative power, to the MMDA. In
the immediate case, it is the City of Makati that possesses the police power capable of compelling the opening of a
private street for public use, as long as there is a lawful subject, and that it be enacted by lawful means.
Binay v. Domingo
[POLICE POWER]

Facts:
The Municipality of Makati enacted Resolution No. 60, which gave financial assistance of five hundred
pesos to a bereaved family within the locality whose gross income does not exceed two thousand pesos a month. The
law further states that the funds therein will be taken from unappropriated funds from the municipal treasury. The
Commission on Audit assailed the resolution, claiming that an ordinance must have a real, substantial, or rational
relation to public health, morals, or general welfare for it to be a valid exercise of police power.

Issues:
1. Is the law a valid exercise of police power?

Ruling:
1. YES. According to the Supreme Court, while COA correctly noted the delegation of police power to
local governments through the general welfare clause, it erred in confining the definition of such power to “public safety
and general welfare”. Public purpose is not unconstitutional merely because it benefits a limited number of persons. The
care for the poor is a public duty, and the government, by virtue of the principle of parens patriae, has the duty of
making sure the poor are attended to. Thus financial support for the less fortunate has long been expected as a valid
exercise of police power aimed at the promotion of the common good.
City of Manila v. Judge Laguio
[POLICE POWER/EQUAL PROTECTION]

Facts:
The City of Manila enacted Ordinance No. 7783, which prohibits the establishment or operation of certain
forms of businesses like sauna parlors, karaoke bars, inns and hotels in the Malate area for the purpose of protecting
the general welfare and morals, as these places are often the fonts of illicit activities within the locality. Malate Tourist
Development Corporation contends that this is an invalid exercise of police power, as it unnecessarily includes inns,
hotels, lodging houses, and motel in the classification.

Issues:
1. Is the law a valid exercise of police power?
2. Does it violate the equal protection clause?

Ruling:
1. NO. An ordinance, to be a valid exercise of police power, needs to conform to the following
substantive requirements:
1) Must not contravene the Constitution or any statute
2) Must not be unfair or oppressive
3) Must not be partial or discriminatory
4) Must not prohibit but may regulate trade
5) Must be general and consistent with public policy
6) Must not be unreasonable

In the case above, it is clear that the ordinance is unreasonable and oppressive, as it substantially divests the
respondent of the beneficial use of its property. Moreover, it does not specify the rules and regulations to be observed,
rather, it exercises an assumed power to prohibit trade. For violating these two essential requirements, the ordinance is
deemed an invalid exercise of police power.

2. YES. The guarantee of equal protection requires that all persons or things similarly situated should be
treated alike, both as to rights conferred and responsibilities imposed. The guarantee means that no person or class of
persons shall be denied the equal protection of laws which is enjoyed by other persons or other classes in like
circumstances. In the immediate case, the MTDC’s businesses were, without basis, classified as grounds for illicit
activities.
White Light Corporation v. City of Manila
[POLICE POWER]

Facts:
Through Ordinance No. 7774, the City of Manila prohibited short-time admission and wash-up rates for
the same in hotels, motels, inns, lodging houses, and similar establishments, in order to minimize the use of these
establishments for illicit activities. Malate Tourist Development Corporation contends that this is an invalid exercise of
police power, as it unreasonably interferes with their business.

Issues:
1. Is the law a valid exercise of police power?

Ruling:
1. NO. The fact that the ordinance deprives patrons of a product and the petitioners with a lucrative
business places the validity of the exercise of police power in question. According to the Supreme Court, aside from the
requirements of lawful subject and lawful means, there should also appear a reasonable relation between the purposes
of the measure and the means employed for its accomplishment, otherwise, what would ensue is arbitrary invasion of
private rights and deprivation of property in the guise of police power.
Acebedo Optical v. Court of Appeals
[POLICE POWER]

Facts:
A city mayor, at the behest of local optometrists, imposed special conditions on the release of a business
permit for Acebedo Optical within the locality. In setting such conditions, he sought to prevent the latter from opening up
an optical clinic and from prescribing and selling reading or optical eyewear.
Issues:
1. Is the law a valid exercise of police power?

Ruling:
1. NO. In general, the issuance of business licenses and permits by a municipality or city is essentially
regulatory in nature. The authority of local government units to issue or grant such licenses or permits is in the exercise
of the police power of the State. However, a business permit is issued to regulate the mere conduct of business within
the city, therefore the mayor cannot regulate the practice of a profession a power vested exclusively within the
Professional Regulation Commission and the Board of Examiners in Optometry in this case. This act is oppressive and
discriminatory, not to mention unreasonable. The mayor could instead have required the optometrists in petitioner’s
business to produce a valid certificate of registration or license.
City Government of Quezon City v. Ericta
[POLICE POWER/EMINENT DOMAIN]

Facts:
Section 9 of Ordinance No. 6118 requires that at least 6% of the total area of the memorial park
cemetery be set aside for charity burial of deceased persons who are paupers and have been citizens of Quezon City
for at least five years prior to their death.

Issues:
1. Is the law a valid exercise of police power?

Ruling:
1. NO. If an owner is deprived of his property by virtue of police power, the property is generally
summarily destroyed in order to promote the general welfare, as in the case of a nuisance per se. In this case, the lots
are instead converted to public use, which is not an exercise of police power but rather, of eminent domain, as there is
a taking of property, without just compensation to boot.
ABS-CBN v. Philippine Multi-Media Services Incorporated
[POLICE POWER/EMINENT DOMAIN]

Facts:
PMSI was granted a legislative franchise under RA 8630 to install, operate, and maintain a nationwide
DTH satellite service. Under the NTC Memorandum Circular No. 4-08-88, it is required to carry the television signals of
authorized television broadcast stations. ABS-CBN sued PMSI alleging, among others, that the aforementioned NTC
Circular is violative of Section 9 of the Bill of Rights, for it allows the taking of private property without just
compensation.

Issues:
1. Is the law a valid exercise of eminent domain ?

Ruling:
1. NO. However, the contention of ABS-CBN is erroneous at best. The carriage of ABS-CBN’s signals by
virtue of the must-carry rule in Memorandum Circular No. 04-08-88 is under the direction and control of the government
though the NTC which is vested with exclusive jurisdiction to supervise, regulate and control telecommunications and
broadcast services/facilities in the Philippines. Therefore, ABS-CBN cannot claim to enjoy private ownership of a
television signal. Moreover, the imposition of the must-carry rule is within the NTC’s power to promulgate rules and
regulations, as public safety and interest may require, to encourage a larger and more effective use of communications,
radio and television broadcasting facilities, and to maintain effective competition among private entities.
Office of the Solicitor General v. Ayala Land Incorporated
[POLICE POWER]

Facts:
The Senate Committee on Trade and Commerce found that the collection of parking fees by shopping
malls is contrary to the National Building Code. Thus the committee recommended that the OSG should enjoin the
collection of parking fees and enforce the sanctions for violation of the National Building Code. Respondents Ayala
Land, Robinsons Land Corporation, Shangri-La Plaza, and SM Prime Holdings, Inc. contended that they could not be
obliged to provide free parking spaces in their malls to patrons and the general public.

Issues:
1. May the OSG prohibit the malls from collecting parking fees as an exercise of police power?

Ruling:
1. NO. Firstly, preventing the collection of fees by mall parking lots is not a regulation of trade, but rather
a prohibition, and an encroachment of the right of private entities over their property. Secondly, police power cannot
degrade to arbitrary or whimsical prohibition, as it should first pass the test of lawful means and lawful subject. Police
power does not involve taking of property, unless there is a need to destroy the same for the protection of peace and
order, and the protection of public welfare.
Manila International Airport Authority v. City of Pasay
[TAXATION]

Facts:
The City of Pasay is looking to impose real estate tax on the Manila International Airport Authority for the
operation of the Ninoy Aquino International Airport. Herein petitioner however, contends that being an instrumentality of
the state, it cannot be subject to taxation.

Issues:
1. May the MIAA be compelled to pay taxes to the City of Pasay?

Ruling:
1. NO. MIAA is a government instrumentality vested with corporate powers to efficiently perform its
function. Therefore, it retains the governmental powers of eminent domain, police authority, and levying of fees and
charges. As such, MIAA is not a government-owned and controlled corporation, and therefore beyond the taxing power
delegated to local government units.
Ouano v. Republic / Mactan Cebu International Airport Authority v. Inocian
[EMINENT DOMAIN]

Facts:
At the center of both cases is the issue regarding the right of reconveyance of the former owners of
private lots appropriated for public use. Both petitions involve separate lots appropriated by the MCIAA for the purpose
of expanding the Lahug International Airport. When the public purpose (Lahug Airport had been abandoned and a
portion of it had in fact been purchased by a private corporation for development as a commercial complex) for which
the properties were appropriated had ceased, petitioners filed to repurchase the land, but the MCIAA contended that
there was no written stipulation that the government had granted such a right to the former owners.

Issues:
1. Should petitioners have a right to repurchase their land once the public purpose for which it was
appropriated under the power of eminent domain cease to exist?

Ruling:
1. YES. Eminent domain is subject to two mandatory limitations:
1) That the property be taken for public use
2) That private owner be given just compensation

Public use now means that there should be a genuine necessity that calls for the expropriation of the
private property. Should this necessity disappear or cease, or if the public use deviates from the declared purpose to
benefit another private person, it should behoove the condemner to return the property to its rightful owner, and the
latter in turn should return just compensation already paid. The government cannot plausibly keep the property it
expropriated in any manner it pleases and, in the process, dishonor the judgment of expropriation. This is not in keeping
with the idea of fair play,
Air Transportation Office v. Spouses Ramos
[EMINENT DOMAIN]

Facts:
Spouses David and Elisea Ramos agreed to convey their land to the government for use by the City of
Baguio, as part of the runway and running shoulder of the Laokan Airport operated by petitioner ATO. However,
petitioner failed to pay despite repeated verbal and written demands. Respondents filed an action for collection, but
petitioner assails that since it acted in an official, and not proprietary manner when it appropriated the land, the state
cannot be sued without its consent.

Issues:
1. Can the state’s immunity from suit defeat the constitutional right to just compensation for expropriation
under eminent domain?

Ruling:
1. NO. It is true that the ATO acted as an agent of the state in the performance of an act jus imperii when
it appropriated the land. However, the state’s immunity from suit cannot be tailored to perpetrate an injustice. The
Constitution , through the Bill of Rights, demands that private property shall not be taken for public use without payment
of just compensation. This compensation serves as payment for the deprivation of the private individual of the beneficial
enjoyment of his property. As such, the defense of immunity from suit cannot be effectively set up by the state against
an action for payment by the owners.
Heirs of Moreno v. Mactan Cebu International Airport
[EMINENT DOMAIN]

Facts:
The city government of Mactan, Cebu appropriated the land of Moreno as part of the expansion plan of
Lahug Airport. When the airport ceased its operation, the heirs of Moreno demanded their right to reconveyance.

Issues:
1. Should the government grant the petition for reconveyance?
2. What are the reciprocal obligations of the state and a former property owner in an act of reconveyance
after taking under eminent domain?

Ruling:
1. YES. When the reason or public purpose for which the property was appropriated ceases to exist, the
state must reconvey the property to the former owners.

2. Expropriation under eminent domain is in the form of a constructive trust contract. Therefore, as the
state is obliged to reconvey the land, the private owner is likewise obliged to return the amount they received as just
compensation with consequential damages. Moreover, the owner is likewise obliged to pay for the improvements made
upon the property, in accordance with Article 1189 of the New Civil Code, which states: “If the thing is improved by its
nature, or by time, the improvement shall inure to the benefit of the creditor.”
Mactan Cebu International Airport v. Lozada
[EMINENT DOMAIN]

Facts:
Deiparine is the owner of a track of land expropriated by the government of Cebu for the expansion
project of Lahug Airport. When the State took control of the various tracks of land for expansion, it made a compromise
with the landowners that should they withdraw their appeals or refrain from it, the expropriated lots would be resold at
the price they were expropriated in in the event that the ATO would abandon the Lahug airport project. During the
pendency of these proceedings, Lozada came into possession of Deiparine’s land. When the expropriation was
approved, Lozada, along with other landowners, received compensation. However, when the project was eventually
abandoned, and herein respondent demanded for his right to reconveyance, the ATO desisted, stating that they had not
expressly granted Lozada the right when the property was expropriated.

Issues:
1. Should Lozada be granted the right to reconveyance?

Ruling:
1. YES. The expropriator who takes property under eminent domain must commit to use the property
pursuant to the purpose stated in the petition filed. Failing this, it should file another petition for the new purpose.
Otherwise, it is incumbent upon said expropriator to return the property to its private owner should the latter choose to
reacquire the same. Moreover, the compromise agreement the government made to the private landowners should be
binding.
Tolentino v. Secretary of Finance
[TAXATION/FREEDOM OF RELIGION/FREEDOM OF THE PRESS/NON-IMPAIRMENT CLAUSE]

Facts:
The value-added tax (VAT) is levied on the sale, barter or exchange of goods and properties as well as
on the sale or exchange of services. RA 7716 seeks to widen the tax base of the existing VAT system and enhance its
administration by amending the National Internal Revenue Code. There are various suits challenging the
constitutionality of RA 7716 on various grounds.

Issues:
1. Does the EVAT violate the freedom of religious institutions from taxation?
2. Does the EVAT violate the tax exemption granted to members of the press?
3. Does the EVAT violate the non-impairment of contract clause?

Ruling:
1. NO. In terms of religious freedom, the VAT is not a license tax, nor one imposed on the exercise of a
privilege (as in selling religious merchandise). Instead, it is imposed on the sale, barter, lease, or exchange of goods or
services purely for revenue purposes. The resulting burden on the exercise of religious freedom is so incidental as to
make it difficult to differentiate it from any other economic imposition that might make the right to disseminate religious
doctrines costly.

2. NO. As a general rule, the press is not exempt from the taxing power of the state. What freedom of the
press prohibits are laws which single out or discriminate against members of the media. Moreover, whenever the
government grants tax exemptions to the press, it is a matter of privilege, not right, and therefore may be waived
without offense to the constitution.

3. NO. Yes. When a lawful tax on a new subject or an increased tax on an old one is imposed, it cannot
be said to interfere with a contract or impairs its obligation within its constitutional meaning. Contracts must be
understood as having been made in reference to the possible exercise of the rightful authority of the government and no
obligation of contract can extend to defeat that authority.
City Government of Quezon City v. Bayan Telecommunications, Inc.
[TAXATION]

Facts:
The city government of Quezon City imposed real property taxes on all real properties in the locality, by
virtue of Congress’ delegation of its taxing power through the Local Government Code. Subsequently, Congress
enacted a statute that granted real property tax exemptions to all telecommunications franchises. Bayantel, being one
such franchise, in firm belief of its exempted status, refused to pay the taxes assessed against it by the city
government. The Quezon City Treasurer on the other hand, sent out notices of delinquency and eventually, several
warrants of levy against Bayantel’s properties in preparation for their sale at a public auction.

Issues:
1. May the City Government compel the respondents to pay real property taxes despite exemptions?

Ruling:
1. NO. The power to impose taxes is primarily lodged in Congress, and may be exercised by local
government units under mandate of the Local Government Code. However, this grant of power to local governments
does not affect the power of the National Legislature to grant exemptions to certain persons, pursuant to a declared
national policy. In a previous case (PLDT v. City of Davao), the Supreme Court upheld the power of Congress to grant
exemptions over the power of the LGUs to impose taxes.
Film Development Council of the Philippines v. Colon Heritage Council Cebu
[TAXATION]

Facts:
Pursuant to Republic Act 9167, the Film Development Council of the Philippines enjoys the right to
collect 100% and 65% of local amusement taxes imposed by local governments and reward these to films it graded “A”
and “B”. When FDCP collected the taxes from Colon Heritage and SM Prime Holdings, Inc., the city government of
Cebu protested, claiming that the provision unduly deprives them of their revenue.

Issues:
1. May the FDCP validly collect taxes from local governments?

Ruling:
1. NO. The Supreme Court deemed this incentive system as a confiscatory measure that infringes on the
budgetary autonomy of local governments. LGUs are granted the constitutional prerogative to apportion their funds as
they deem fair and equitable.
Batangas City et al v. Pilipinas Shell
[TAXATION]

Facts:
In 2002, the City of Batangas issued a notice of collection of business tax to Pilipinas Shell Petroleum
Corporation for its operation of an oil refinery in Tabagao, Batangas. The latter desisted, claiming that it is not liable to
pay a local business tax either as manufacturer or distributor of petroleum products.

Issues:
1. May the local government of Batangas validly levy taxes on the operations of a petroleum
manufacturer and distributor?

Ruling:
1. NO. Although the power to tax is inherent in the state, the same is not true for local governments,
because the constitutional delegation of such power to LGUs is not as all-encompassing as it is with the National
Legislature. Article X, Section 5 of the 1987 charter provides: “Each local government unit shall have the power to
create its own sources of revenues and to levy taxes, fees, and charges subject to such guidelines and limitations as
the Congress may provide, consistent with the basic policy of local autonomy. Such taxes, fees, and charges shall
accrue exclusively to the local governments.”
Moreover, Section 130 of the Local Government Code of 1991 provides for the following fundamental
principles governing the taxing powers of LGUs:
1. Taxation shall be uniform in each LGU.
2. Taxes, fees, charges and other impositions shall:
a. be equitable and based as far as practicable on the taxpayer's ability to pay;
b. be levied and collected only for public purposes;
c. not be unjust, excessive, oppressive orconfiscatory;
d. not be contrary to law, public policy, national economic policy, or in the restraint of trade.
3. The collection of local taxes, fees, charges and other impositions shall in no case be left to any private person.
4. The revenue collected pursuant to the provisions of the LGC shall inure solely to the benefit of, and be subject to the
disposition by, the LGU levying the tax, fee, charge or other imposition unless otherwise specifically provided by the
LGC.
5. Each LGU shall, as far as practicable, evolve a progressive system of taxation.
Nursery Care Corporation v. Acevedo & City of Manila
[TAXATION]

Facts:
Petitioners allege double taxation, in that the even as the City of Manila assessed and collected business
taxes from them pursuant to section 15 and 17 of the Revenue Code of Manila, the latter also imposed additional taxes
upon them as a precondition for renewal of their business licenses pursuant to the same Code. Both taxes were
imposed during the same fiscal year.

Issues:
1. Was there double taxation in this case?

Ruling:
1. YES. Double taxation means taxing the same property twice when it should only be taxed once. For
double taxation to exist, the two taxes must be imposed (1) on the same subject matter, (2) for the same purpose, (3)
by the same taxing authority (4) within the same jurisdiction, (5) during the same taxing period, and they must be (6) of
the same character. In the immediate case, both taxes are imposed on the same privilege of doing business within the
city of Manila contributing to city revenues, by the same taxing authority, within the jurisdiction of the City of Manila, for
the same taxing period, and the same character—a local business tax imposed on gross sales or receipts of the
businesses.
People v. Marti
[SEARCHES AND SEIZURES]

Facts:
Appellant and his common-law wife, Sherly Reyes, went to the booth of the Manila Packing and Export
Forwarders carrying four wrapped packages. The appellant informed Anita Reyes that he was sending the packages to
a friend in Zurich, Switzerland. Reyes asked if she could examine and inspect the packages. She refused and assures
her that the packages simply contained books, cigars, and gloves. Before the delivery of appellant’s box to the Bureau
of Customs and Bureau of Posts, the company owner of MPEF, following the standard operating procedure, opened the
boxes for final inspection. A peculiar odor emitted from the box and that the gloves contain dried marijuana leaves. He
then called for the NBI to conduct investigation and laboratory tests.

Issues:
1. May the evidence obtained be admissible?

Ruling:
1. YES. The Supreme Court held that the constitutional prohibition against unlawful searches and
seizures cannot be made to apply to the acts of private individuals. It was the forwarding company’s owner, acting in
accordance with his duties, who opened the box in his place of business. When the NBI was called unto the scene, the
package was already opened, and therefore the act falls under observation in plain sight. This does not make the act of
the NBI a search, much less one prohibited under Section 2 of the Bill of Rights.
Zulueta v. Court of Appeals
[PRIVACY OF COMMUNICATION AND CORRESPONDENCE]

Facts:
Cecilia Zulueta is the wife of Dr. Alfredo Martin. On 26 March 1982, Zulueta entered the clinic of her
husband, a doctor of medicine, and in the presence of her mother, a driver and Martin’s secretary, forcibly opened the
drawers and cabinet in her husband’s clinic and took 157 documents consisting of private correspondence between Dr.
Martin and his alleged paramours, greetings cards, cancelled checks, diaries, Dr. Martin’s passport, and photographs.
The documents and papers were seized for use in evidence in a case for legal separation and for disqualification from
the practice of medicine which Zulueta had filed against her husband. Dr. Martin brought the action for recovery of the
documents and papers and for damages against Zulueta, with the Regional Trial Court of Manila, Branch X.

Issues:
1. May the evidence obtained be admissible?

Ruling:
1. NO. The constitutional safeguard against violations of the privacy of communications and
correspondence is no less applicable simply because it is the wife who is the party against whom the constitutional
provision is to be enforced. The second paragraph of Section 3 of the Bill of Rights provides the only valid exception to
this rule, to wit: when there is a lawful order of a court, or when public safety or order requires otherwise, as provided by
law. The intimacies between husband and wife do not justify any one of them in breaking the drawers and cabinets of
the other and in ransacking them for any telltale evidence of marital infidelity.
Yrasuegui v. Philippine Airlines
[EQUAL PROTECTION]

Facts:
Yrasuegui was a former international flight steward of Philippine Airlines. He was dismissed because of
his failure to comply with the weight standards of the airline company after making multiple promises to adhere to the
same. Petitioner claims his dismissal infringed on his right to equal protection of the laws, because he was
discriminated against for his weight.

Issues:
1. Was the dismissal violative of the equal protection clause?

Ruling:
1. NO. In the absence of governmental interference, the liberties granted by the Constitution cannot be
invoked. The Bill of Rights in particular, cannot be invoked against the acts of private individuals. Indeed, the US
Supreme Court, in interpreting the 14th Amendment, which is the source of our equal protection guarantee, is
consistent in saying that the equal protection erects no shield against private conduct, however discriminatory or
wrongful. Private actions, no matter how egregious, cannot violate the equal protection guarantee.
Ang Tibay v. Court of Industrial Relations
[DUE PROCESS]

Facts:
Teodoro Toribio owns and operates Ang Tibay, a leather company.Due to alleged shortage of leather,
Toribio caused the layoff of members of National Labor Union (NLU). NLU averred that Toribio’s act is not valid. The
CIR, decided the case and elevated it to the SC, but a motion for new trial was raised by the NLU. But Ang Tibay filed a
motion for opposing the said motion.

Issues:
1. Can the CIR, as a special court vested with more administrative than judicial functions, dispense with
due process?

Ruling:
1. NO. The CIR is more an administrative board than a part of the integrated judicial system of the
nation. Its functions are far more comprehensive and extensive. As such, it is free from rigidity of certain procedural
requirements, but this does not mean that it can disregard a person’s right to due process. Administrative due process,
in fact, requires that:

1)The accused be given the right to a hearing, which includes the right to present one’s cause and submit evidence in
support thereof.
2)The tribunal must consider the evidence presented
3)The decision must have something to support itself
4)The evidence must be substantial
5)The decision must be based on the evidence presented at the hearing, or at least contained in the record and
disclosed to the parties affected
6)The tribunal or body or any of its judges must act on its own independent consideration of the law and facts of the
controversy, and not simply accept the views of a subordinate
7)The board or body should, in all controversial questions, render its decision in such a manner that the parties to the
proceeding can know the various issues involved, and the reason for the decision rendered.
Quisumbing v. Rosales
[DUE PROCESS]

Facts:
On September 18, 2013, a meeting was held at the Commission on Human Rights (CHR) to discuss the
complaints of several CHR employees against Commissioner Cecilia Rachel Quisumbing, among which includes
maltreatment of her employees, and the crimes of malfeasance or misfeasance. Present during the meeting were
Commissioners Etta Rosales, Ma. Victoria Cardona, and Norberto dela Cruz. Commissioner Quisumbing was on sick
leave while Commissioner Mamauag was on official business. On the basis of these complaints and affidavits, the
Commissioners present issued a Resolution and a Show Cause order requesting Comm. Quisumbing to submit within 5
days a written explanation why she should not be held administratively liable for any administrative liability, and to
transmit her explanation to the Office of the Ombudsman.

Issues:
1. Was the petitioner denied due process when the CHR forwarded her case to the Ombudsman without
giving her the benefit of a hearing?

Ruling:
1. NO. Firstly, the CHR in this case did not act in a judicial or even quasi-judicial capacity. It merely
conducted an inquiry proceedings in the manner of a fact-finding investigation, which is not an adjudication which
warrants a formal hearing and trial. Moreover, the petitioner was not denied due process, because she was properly
apprised of the allegations against her, and she was even given ample opportunity to hear her side. This is clearly in
line with the spirit of administrative due process, which states that every person must be given a chance to explain his
side, or to seek a reconsideration of the accusations complained of.
Biraogo v. Philippine Truth Commission
[EQUAL PROTECTION]

Facts:
Pres. Aquino signed E. O. No. 1 establishing Philippine Truth Commission of 2010 (PTC) dated July 30,
2010. PTC is an ad hoc body formed under the Office of the President with the primary task to investigate reports of
graft and corruption committed by third-level public officers and employees, their co-principals, accomplices and
accessories during the previous administration, and to submit its finding and recommendations to the President,
Congress and the Ombudsman.

Issues:
1. Does this law violate the equal protection clause?

Ruling:
1. YES. Equal protection requires that all persons or things similarly situated should be treated alike, both
as to rights conferred and responsibilities imposed. It requires public bodies and institutions to treat similarly situated
individuals in a similar manner. This Order must be struck down as a violation of the equal protection clause. The clear
mandate of EO No. 1 is to investigate cases of graft and corruption only in the previous administration, which unduly
lays a burden on the Arroyo government, where the same cannot be said of the other members of the class of other
administrations. For a law to be within the ambit of equal protection, it needs to pass the reasonableness test, with the
following requisites:
1)The classification must rest on substantial distinctions
2)It must be germane to the purposes of the law
3)It should not be limited to existing conditions only
4)It should apply equally to all members of the same class
Fariñas v. Executive Secretary
[EQUAL PROTECTION]

Facts:
Petitioners assail that Section 14 of Republic Act No. 9006, otherwise known as the Fair Election Act, is
unconstitutional, because it violates the equal protection clause, as it discriminates against appointive officials. In the
Omnibus Election Code, both elective and appointive officials are considered ipso facto resigned from their office once
they file a certificate of candidacy. The assailed section of RA 9006 expressly repeals this provision in favor of elective
officials, but it is implied that the election law provision still applies to appointive officials.

Issues:
1. Does this law violate the equal protection clause?

Ruling:
1. NO. Substantial distinctions clearly exist between elective officials and appointive officials. The former
occupy their office by virtue of the mandate of the electorate. They are elected to an office for a definite term and may
be removed therefrom only upon stringent conditions. On the other hand, appointive officials hold their office by virtue of
their designation thereto by an appointing authority. Some appointive officials hold their office in a permanent capacity
and are entitled to security of tenure while others serve at the pleasure of the appointing authority.
Quinto v. Commission on Elections
[EQUAL PROTECTION]

Facts:
Petitioners Eleazar P. Quinto and Gerino A. Tolentino, Jr. filed a petition for certiorari and prohibition
against the COMELEC for issuing a resolution declaring appointive officials who filed their certificate of candidacy as
ipso facto resigned from their positions. Two of the justifications implied from such provisions are: a) to prevent the use
of appointive positions in the promotion of one’s candidacy, and b) to prevent neglect and inefficiency which could result
from focusing on the official’s campaign instead of his official functions.

Issues:
1. Does this law violate the equal protection clause?

Ruling:
1. YES. Substantial distinctions aside, the provision is clearly in not germane to the law. In accepting the
two reasons why this law was passed, one can surmise that prohibiting appointive officials from holding office while
campaigning is reasonable. Yet there is no palpable reason why the same dangers sought to be prevented cannot exist
in elected officials. For example, the Executive Secretary could wield the same influence as the Vice-President who at
the same time is appointed to a Cabinet post. With the fact that they both head executive offices, there is no valid
justification to treat them differently when both file their CoCs for the elections. In considering persons holding
appointive positions as ipso facto resigned from their posts upon the filing of their CoCs, but not considering as resigned
all other civil servants, specifically the elective ones, the law unduly discriminates against the first class. The fact alone
that there is substantial distinction between those who hold appointive positions and those occupying elective posts,
does not justify such differential treatment.

Important note:
The Supreme Court in this case abandoned their previous ruling in the Fariñas case. However, they held that the
discussion on the previous case regarding the equal protection clause constitutes an orbiter dictum, particularly on the
topic of substantial distinction, which may nevertheless be used in reference.
Duncan Association of Detailman-PTGWO v. Glaxo Wellcome Philippines
[EQUAL PROTECTION]

Facts:
Tecson took a job as a medical representative for Glaxo in 1995. Part of his contract of employment is a
stipulation wherein he agrees to study and abide by the existing company rules, and to disclose to management any
existing future relationship by consanguinity or affinity with co-employees or employees with competing drug
companies. The stipulation also provides that he should resign should the relationship pose a possible conflict of
interest. Tecson was soon assigned to Camariñes Norte where he fell in love with an employee from a competitor firm.
When Glaxo found out, they sought to reassign him, wherein he filed a petition with the Court , claiming that he was
unduly discriminated against.

Issues:
1. Does this law violate the equal protection clause?

Ruling:
1. NO. A prohibition against personal or marital relationship among employees is reasonable in the case
at bar, especially considering the case at bar. Glaxo has a right to guard its trade secrets, manufacturing formulas,
marketing strategies, and other confidential programs and information from competitors. Furthermore, it is a settled
principle that the safeguards of the equal protection clause only applies where the State is involved, or where a private
individual or public officer acts in the guise of state interest or function.
PT&T v. NLRC and Grace de Guzman
[DUE PROCESS/EQUAL PROTECTION]

Facts:
Grace de Guzman is a probationary employee of PT&T. In her job application, she represented that she
was single although she was married. When management found out, she was made to explain. However, her
explanation was found unsatisfactory so she was subsequently dismissed from work.

Issues:
1. Was the dismissal valid?

Ruling:
1. NO. The real issue at bar in this case, as the Supreme Court found out, is that the company seems to
discriminate against women laborers, as it sought the dismissal of married women. Acknowledged as paramount in the
due process scheme is the constitutional guarantee of protection to labor and security of tenure. Thus, an employer is
required, as a condition sine qua non prior to severance of the employment ties of an individual under his employ, to
convincingly establish the existence of a valid and just cause in dispensing with the services of such employee, ones
labor being regarded as constitutionally protected property. While the regulation of manpower might fall under the ambit
of a company’s management prerogatives, unlawful discrimination is repugnant to our country’s labor laws, and even
the Constitution itself, as enshrined in the due process and equal protection clauses.
International School Alliance of Educators v. Quisumbing
[EQUAL PROTECTION]

Facts:
International School pays its foreign-hired teachers a higher salary than its local-hires, whether the latter
are Filipino or not. The institution justifies this policy under the ‘dislocation factor’ – that foreigners must be given a
higher salary both to attract them to teach here, and to compensate them for the “significant economic disadvantages”
involved in coming here. The Teacher’s Union assails this as an act discriminatory against local teachers.

Issues:
1. Is there a violation of the equal protection clause?

Ruling:
1. YES. While it is important for the school to attract foreign-hires, salary should nevertheless be used to
the prejudice of the local-hires. The local-hires perform the same services as foreign-hires and they ought to be paid the
same salaries as the latter. For the same reason, the "dislocation factor" and the foreign-hires' limited tenure also
cannot serve as valid bases for the distinction in salary rates. The dislocation factor and limited tenure affecting foreign-
hires are adequately compensated by certain benefits accorded them which are not enjoyed by local-hires, such as
housing, transportation, shipping costs, taxes and home leave travel allowances. For these reasons, the classification
made by IS should be struck down as it is not made of substantial distinction.
Garcia v. Judge Drilon
[EQUAL PROTECTION]

Facts:
Jesus Garcia, after his wife filed a case against him (claiming physical, psychological, emotional, and
economic violence), assails the constitutionality of Republic Act No. 9262, or the Violence Against Women and their
Children (VAWC) Law.

Issues:
1. Is the VAWC law violative of the equal protection clause?

Ruling:
1. NO. Equal protection simply requires that all persons or things similarly situated should be treated
alike, both as to rights conferred and responsibilities imposed. All that is required of a valid classification is that it be
reasonable, which means that the classification should be based on substantial distinctions which make for real
differences; that it must be germane to the purpose of the law; not limited to existing conditions only; and apply equally
to each member of the class. There is no question as to the validity of the classification of women and children as
victims more often than violators of acts of abuse. Moreover, the legislative intent in passing the law is just as clear, the
increased judicial protection of women and children in cases of abuse being the most logical and least destructive
method of attaining the legislature’s goal.
1-UTAK v. COMELEC
[EQUAL PROTECTION/FREEDOM OF EXPRESSION]

Facts:
Resolution No. 9615 provides for the rules implementing R.A. No. 9006 in connection with the May 13,
2013 national and local elections. Section 7, particularly subsections 5(f) and 6(f), prohibited the posting or displaying of
campaign propaganda in public utility vehicles such as buses, jeepneys, trains, taxis, pedicabs, and tricycles,as well as
in public transport terminals. Petitioner sought the invalidation of these provisions on the ground of violation of the equal
protection clause.

Issues:
1. Did the aforementioned provisions violate the right of private owners of PUVs to equal protection of the
laws?

Ruling:
1. YES. The aforementioned provisions carry with them the penalty of revocation of the public utility
franchise and shall make the owner thereof liable for an election offense. The prohibition constitutes a clear prior
restraint on the right to free expression of the owners of PUVs and transport terminals. As a result of the prohibition,
owners of PUVs and transport terminals are forcefully and effectively inhibited from expressing their preferences under
the pain of indictment for an election offense and the revocation of their franchise or permit to operate. Moreover, there
is no substantial distinction between owners of PUVs and transport terminals and owners of private vehicles and other
properties. Ownership of PUVs and transport terminals, though made available for use by the public, remains private. If
owners of private vehicles and other properties are allowed to express their political ideas and opinion by posting
election campaign materials on their properties, there is no cogent reason to deny the same preferred right to owners of
PUVs and transport terminals.
Central Bank Employees v. Bangko Sentral
[EQUAL PROTECTION]

Facts:
Section 15 (i), Article II of RA 7653 establishes a Human Resource Management System and a
compensation structure as part of the authority of the Monetary Board. Employees whose positions fall under SG 19
and below shall be in accordance with the rates in the salary standardization act. Petitioner contends that the
classifications is not reasonable, arbitrary and violates the equal protection clause. The Solicitor General, as counsel of
the Executive Secretary defends the provision, that the classification of employees is based on real and actual
differentiation and it adheres to the policy of RA 7653 to “establish professionalism and excellence within the BSP
subject to prevailing laws and policies of the government.”

Issues:
1. Does this provision violate the equal protection clause?

Ruling:
1. YES. The proviso is unconstitutional as it operate on the salary grade or the officer employee status, it
distinguishes between economic class and status with the higher salary grade recipients are of greater benefit above
the law than those of mandated by the Salary Standardization Act. Officers of the BSP receive higher wages that those
of rank-and-file employees because the former are not covered by the salary standardization act as provided by the
proviso.
Republic of the Philippines v. Pilipinas Shell Petroleum Corporation
[DUE PROCESS]

Facts:
On 10 October 1984, the Oil Price Stabilization Fund (OPSF) was created under Presidential Decree No.
1956 for the purpose of minimizing frequent price changes brought about by exchange rate adjustments and/or increase
in world market prices of crude oil and imported petroleum products. On December 1991, the Office of Energy Affairs,
now the DOE, informed the respondent corporation that its contributions to the OPSF were insufficient. Thus, in
compliance with MOF Circular No. 1-85, the DOE imposed a surcharge of ₱14,414,860.75. Respondent averred that
the law is in violation of the right to due process because it was not published in the Official Gazette or a newspaper of
general circulation.

Issues:
1. Is the surcharge in violation of due process?

Ruling:
1. YES. As early as 1986, the Supreme Court in Tañada v. Tuvera enunciated that publication is
indispensable in order that all statutes, including administrative rules that are intended to enforce or implement existing
laws, attain binding force and effect. In the immediate case, the petitioner was unable to convince the Court that there
has been publication in accordance with the Civil Code and the Administrative Code. The due process requirements of
publication and filing cannot be set aside, even when the assailed law is an administrative circular or an implementing
rules and regulations.
Republic of the Philippines v. Judge Eugenio
[EX POST FACTO LAWS]

Facts:
Under the authority granted by the Resolution, the AMLC filed an application to inquire into or examine
the deposits or investments of Alvarez, Trinidad, Liongson and Cheng Yong before the RTC of Makati, Branch 138. Lilia
Cheng Yong argues that the AMLC cannot have a retroactive effect, it being a purely substantive penal statute.
Therefore, the bank inquiry order could not apply to deposits or investments opened prior to the effectivity of Rep. Act
No. 9164, or on 17 October 2001. Thus, she concludes, her subject bank accounts, opened between 1989 to 1990,
could not be the subject of the bank inquiry order lest there be a violation of the constitutional prohibition against ex post
facto laws.

Issues:
1. Is the contention of Lila Cheng correct?

Ruling:
1. YES. No ex post facto law may be enacted, and no law may be construed in such fashion as to permit
a criminal prosecution offensive to the ex post facto clause. As applied to the AMLA, it is plain that no person may be
prosecuted under the penal provisions of the AMLA for acts committed prior to the enactment of the law on 17 October
2001. As much was understood by the lawmakers since they deliberated upon the AMLA, and indeed there is no
serious dispute on that point. An ex post facto law is one which either:
1)Makes criminal an act done before the passage of the law and which was innocent when done, and punishes such an
act,
2)Aggravates a crime, or makes it greater than it was when it was committed
3)Changes the punishment and inflicts a greater punishment than the law annexed to the crime when committed
4)Alters the legal rules of evidence, and authorizes conviction upon less or different testimony than the law required at
the time of the commission of the offense
5)Assuming to regulate civil rights and remedies only, in effect imposes penalty or deprivation of a right for something
which when done was lawful, and
6)Deprives a person accused of a crime of some lawful protection to which he has become entitled, such as the
protection of a former conviction or acquittal, or a proclamation of amnesty.
Prudente v. Dayrit
[SEARCHES AND SEIZURES]

Facts:
Major Alladin Dimagmaliw, chief of the Intelligence Special Action Division (ISAD) of the Western Police
District, filed with Judge Abelardo Dayrit of the Manila RTC for the issuance of a search warrant for violation of the
Illegal Possession of Firearms. The information goes as follows: That in PUP Dr. Nemesis Prudente has in his control or
possession firearms, explosives, hand grenades and ammunition which are illegally possessed or intended to be used
as means for the commission of an offense, that the undersigned has verified the report and found it to be factual, and
therefore, believes that a search warrant should be issued. Respondent judge issued the warrant upon these merits.

Issues:
1. Was the issuance of the search warrant valid?

Ruling:
1. NO. The Supreme Court, in Alvarez v. CFI, stated that an application for search warrant or in a
supporting deposition must refer to the truth of the facts within the personal knowledge of the applicant and/or his
witnesses, and not merely on facts reported by a person whom he considers to be reliable. In the case at bar, the
constitutional mandate that a warrant shall not issue except upon personal determination of probable cause by the
judge is lacking. Moreover, the warrant failed to particularly describe the place to be searched, as there were several
rooms on the ground floor and the second floor of PUP. Most damning however, is the blatant failure of the complainant
to establish probable cause to justify the issuance of a search warrant.
20th Century Fox v. Court of Appeals
[SEARCHES AND SEIZURES]

Facts:
20th Century Fox Film Corporation sought the assistance of the NBI in conducting searches and
seizures in connection with the NBI’s anti-film piracy campaign. Petitioner alleged that certain videotape outlets all over
Metro Manila are engaged in the unauthorized sale and renting out of copyrighted films in violation of PD No. 49 (the old
Intellectual Property Law). After conducting surveillance and obtaining witness testimonies, the NBI filed three
applications for search warrants, which was subsequently issued by the lower court. When private respondents, owners
of the video outlets raided, filed a motion to lift the warrants on the seized property on lack of probable cause, the lower
court granted the petition. CA subsequently affirmed the trial court’s ruling.

Issues:
1. How should probable cause be determined?

Ruling:
1. According to the Supreme Court, the right to privacy of one’s person and papers, which extends to the
right against unlawful searches and seizures, should not be disturbed except in case of overriding social need, and
even then, only under stringent procedural safeguards. Such protections are embodied in the constitutional mandate
that no warrant shall issue except upon no less than the personal knowledge of probable cause by the complainant, as
determined by the judge. It should be noted that it is the personal knowledge of the complainant and his witnesses (the
NBI), and not that of the person seeking the warrant (20 th Century Fox) that is material. In the immediate case, the trial
court was justified in lifting the warrants granted for lack of probable cause.
Sony Music Entertainment v. Judge Español
[SEARCHES AND SEIZURES]

Facts:
Sony Music Entertainment seeks to invalidate the quashal made by Judge Dolores Español on two
search warrants on the ground of lack of probable cause. The warrants, issued in favor of Sony, had in its information,
that private respondents’ business, Solid Laguna Corporation, was in the business of the unlicensed replication,
reproduction, and distribution of videograms without authority from the Video Regulatory Board. The information also
listed an unnamed informant of the NBI pointing to the manufacturing place somewhere in Laguna. Eventually, it was
shown that the court was misinformed about the purported unauthorized reproduction of videos, as the private
respondents were able to furnish a copy of their VRB certificate. Thus, the warrant was set aside.

Issues:
1. Did the judge abuse her discretionary powers in quashing the information and invalidating the
warrants?

Ruling:
1. NO. A core requisite before a warrant shall validly issue is the existence of a probable cause, meaning
the existence of such facts and circumstances which would lead a reasonably discreet and prudent man to believe that
an offense has been committed and that the objects sought in connection with the offense are in the place to be
searched. When the law speaks of facts, the reference is to facts, data or information personally known to the applicant
and the witnesses he may present. In the case at bar, the NBI agents and their witnesses were shown to have no
personal knowledge that the discs in their possession were actually infringing, pirated, or unauthorized copies.
Sammy Malacat v. Court of Appeals
[SEARCHES AND SEIZURES / MIRANDA DOCTRINE]

Facts:
In response to bomb threats made a few days earlier, several police officers were patrolling the streets of
Quiapo, Manila in full uniform. They chanced upon two groups of Muslim-looking men, both acting suspiciously and
eyes looking around very fast, posted at opposite sides of the corner of Quezon Boulevard.. The police officers
positioned themselves in strategic locations and observed for thirty minutes. When the police officers finally confronted
the groups, they fled, and upon giving chase, the police captured petitioner. Upon searching him, the police found a
fragmentation grenade tucked inside his waistline. During custodial investigation, without the presence of counsel,
petitioner admitted to the possession of the grenade recovered. The RTC eventually ruled that albeit the arrest was
made without a warrant, it was akin to a stop-and-frisk, and that the search was incidental to a lawful arrest.

Issues:
1. Was there a valid warrantless arrest in this case?
2. Is the admission of petitioner during custodial investigation admissible in evidence?

Ruling:
1. NO. The Supreme Court offers three reasons for invalidating the arrest (and the subsequent search,
as consequence) made as a stop-and-frisk:
1)On mere observation alone, there is no concrete evidence to suggest that the groups gathered in the aforementioned
corner were the members of the group who attempted to bomb the Plaza Miranda days before.
2)Second, there was nothing in petitioners behavior or conduct which could have reasonably elicited even mere
suspicion other than that his eyes were moving very fast an observation, an incredulous observation given that the
police officers were nowhere near the petitioner when the same was made.
3)Third, there was at all no ground, probable or otherwise, to believe that petitioner was armed with a deadly weapon.
The fact that the weapon was concealed in the waistline suggested that it could not have been immediately visible.

2. NO. The investigation was conducted on the day following his arrest, where no PAO
lawyer was available, and neither was petitioner able to request for one. Thus, even if petitioner consented to the
investigation and waived his rights to remain silent and to counsel, the waiver was invalid as it was not in writing, neither
was it executed in the presence of counsel.

Important note:
There is so much more to this case than what is digested here. It’s a very interesting case, I suggest reading it in full. 
People v. Mengote
[SEARCHES AND SEIZURES]

Facts:
The Western Police District, upon receipt of an information that there were three suspicious persons at
the corner of Juan Luna and North Bay Boulevard in Tondo, Manila, dispatched a surveillance team of plainclothesmen.
The patrolmen looked that the suspicious persons were looking from side to side, one of whom was holding his
abdomen. They approached the persons and identified themselves as policemen, whereupon the two tried to run but
were unable to escape because the other lawmen surrounded them. The suspects were then searched, whereupon
a .38 caliber pistol with live ammunitions was found in the possession of the accused. The weapons were taken from
them and they were turned over to the police headquarters for investigation. An information was filed before the RTC
convicting the accused of illegal possession of firearm.

Issues:
1. Was there a valid warrantless arrest in this case?

Ruling:
1. NO. According to Section 5 of Rule 113, Rules of Court, an arresting officer may validly arrest without
lawful warrant when:
a)The person to be arrested has committed, is actually committing, or is attempting to commit an offense in the
presence of the arresting officer
b)An offense has just been committed, and the arresting officer has personal knowledge of facts indicating that the
person arrested has committed it, and
c)The person to be arrested has escaped from a penal establishment where he is serving final judgment or temporarily
confined while his case is pending, or when he has escaped whilebeing transferred from one place of confinement to
another.
In the case at bar, these requirements have not been sufficiently complied with. At the
time of the arrest in question, the accused appellant was merely looking from side to side and holding his abdomen,
according to the arresting officers themselves. There was apparently no offense that has just been committed or was
being actually committed or at least being attempt by Mengote in their presence. Moreover a person may not be
stopped and frisked in a broad daylight or on a busy street on unexplained suspicion.
People v. Claudio
[SEARCHES AND SEIZURES]

Facts:
Obiña, a police officer, was riding a bus bound to Olongapo from Baguio, and he was seated at the
second to last seat. Anita Claudio alighted the same bus, and while she placed a bag full of vegetables on the last seat
(behind Obiña), she sat on the seat in front of him. Sensing something was amiss, the police officer, reached into the
bag, and smelled the scent of marijuana. The scent was familiar to him, because he was just assigned to the narcotics
division. When the bus stopped at Olongapo, he arrested Claudio and took her into custody for violation of R.A. 6425, or
the Dangerous Drug Act.

Issues:
1. Did Obiña validly exercise a warrantless arrest?

Ruling:
1. YES. The strange behavior of Claudio warranted the search of her bag, so Obiña was justified in doing
just that. Moreover, she was captured in flagrante delicto, because the Dangerous Drugs act punishes the mere
possession, transport, and delivery of prohibited narcotics, therefore she was in fact already committing an offense.
People v. Aminudin
[SEARCHES AND SEIZURES]

Facts:
Acting on a tip from their informants made several days ago, several PC officers arrested Idel Aminnudin
as he was disembarking from the M/V Wilcon 9 at about 8:30 p.m. in Iloilo City. When they searched his bag, they
found what looked like marijuana leaves. They then took him under custody and charged for violation of the Dangerous
Drugs Act.

Issues:
1. Was there a valid warrantless arrest in this case?

Ruling:
1. NO. For one, defendant was not caught in flagrante delicto. When he was accosted, he was merely
disembarking from the vessel, so there was no indication that he might have been committing an offense. Nevertheless,
the most important key to the dismissal of this case is the negligence of the PC officers in disregarding the need to
obtain a valid arrest warrant before arresting the accused. They had to wait two days from the time when they were
informed of a potential crime to the actual arrest, so there is no conceivable reason why they could not have secured a
warrant first.
People v. Tangiblen
[SEARCHES AND SEIZURES]

Facts:
Two patrolmen were conducting surveillance operations at the San Fernando Victory Liner Terminal
when they noticed a person with a traveling bag acting suspiciously. When they confronted him, they then searched his
bag, where they found marijuana leaves. The RTC found the defendant guilty for violating the Dangerous Drugs Act.

Issues:
1. Was there a valid warrantless arrest in this case?

Ruling:
1. YES. One of the exceptions to the general rule requiring a search warrant is a search incident to a
lawful arrest. Moreover, a police officer may, without warrant, arrest a person when, in his presence, the person to be
arrested has committed, is actually committing, or is attempting to commit a crime. Accused was caught in flagrante,
since he was carrying marijuana at the time of his arrest. This case therefore falls squarely within the exception. The
warrantless search was incident to a lawful arrest and is consequently valid.

Important note:
As to the disparity between this and the Aminnudin case, the Supreme Court noted that the case at bar presents
urgency not present in the former. “Although the trial court's decision did not mention it, the transcript of stenographic
notes reveals that there was an informer who pointed to the accused-appellant as carrying marijuana. (TSN, pp. 52-53)
Faced with such on-the-spot information, the police officers had to act quickly. There was not enough time to secure a
search warrant.”
People v. Malmstedt
[SEARCHES AND SEIZURES]

Facts:
Acting on persistent reports that vehicles coming from Sagada were transporting marijuana and other
prohibited drugs, Captain Alen Velasco, the Commanding Officer of the First Regional Command (NARCOM), ordered
his men to set up a temporary checkpoint at Kilometer 14, Acop, Tublay, Mountain Province for inspection of vehicles
coming from the Cordillera Region. Soon, officers entered the bus where Malmstedt, the only Caucasian on board, was
seated. Upon noticing a bulge on his waist, and fearing it might be a weapon, they asked him for his passport and other
identification papers, to which accused failed to comply. The bulging object turned out to be a pouch bag and when
accused opened the same bag, as ordered, the officer noticed four packs of hashish, a derivative of marijuana,
wrapped in brown packing tape. More of the narcotics were found in his luggage, hidden inside teddy bears. Malmstedt
claims illegal search in his defense.

Issues:
1. Was there a valid warrantless search in this case?

Ruling:
1. YES. This case falls within the purview of an arrest in flagrante delicto, because the mere act of
carrying or transporting a prohibited drug is already in violation of the Dangerous Drugs Act. Thus, the search made
upon his personal effects falls squarely under paragraph (1) of Section 5 of the Rules on Criminal Procedure, which
allows a warrantless search incident to a lawful arrest. While it is true that the NARCOM officers were not armed with a
search warrant when the search was made over the personal effects of accused, however, under the circumstances of
the case, there was sufficient probable cause for said officers to believe that accused was then and there committing a
crime.
People v. De Gracia
[SEARCHES AND SEIZURES]

Facts:
In the early morning of December 1, 1989, in the height of the coup d'état staged by ultra-rightist
elements against the government, members of the NCR Defense Command were conducting surveillance of the
Eurocar Sales Office at EDSA, Quezon City, pursuant to an intelligence report stating that the establishment was being
occupied by adverse elements as a command post. After surveying the area for a while, a group of five men
approached the surveillance team, drew their firearms, and fired upon the officers. In retaliation, at around 6 a.m. on
December 5, a search team infiltrated and conducted a raid on the said establishment, wherein they were able to find
and confiscate rifle ammunition, sticks and bundles of dynamite and C-4, and molotov bombs, among others. De
Gracia, one of the people arrested by reason of the raid, pleaded that the evidence gathered be rendered inadmissible
for illegal search.

Issues:
1. Was there a valid warrantless search in this case?

Ruling:
1. YES. The arrest of persons involved in the rebellion whether as its fighting armed elements, or for
committing non-violent acts but in furtherance of the rebellion, is more an act of capturing them in the course of an
armed conflict, to quell the rebellion, than for the purpose of immediately prosecuting them in court for a statutory
offense. The arrest, therefore, need not follow the usual procedure in the prosecution of offenses which requires the
determination by a judge of the existence of probable cause before the issuance of a judicial warrant of arrest and the
granting of bail if the offense is bailable. Obviously the absence of a judicial warrant is no legal impediment to arresting
or capturing persons committing overt acts of violence against government forces, or any other milder acts but really in
pursuance of the rebellious movement. The arrest or capture is thus impelled by the exigencies of the situation that
involves the very survival of society and its government and duly constituted authorities.
Valeroso v. Court of Appeals
[SEARCHES AND SEIZURES]

Facts:
By virtue of a duly issued warrant of arrest, petitioner Valeroso was arrested for a count of kidnapping for
ransom. When he was searched incident to the arrest, a firearm with live ammunition was found tucked by his waist.
The defense claimed that Valeroso was arrested and searched (without a search warrant) in the boarding house of his
children. They pointed their guns on him and tied him and pulled him out of the room as the raiding team went back
inside, searched and ransacked the room. Later, an operative came out of the room exclaiming that he has found a gun
inside. The firearm according to the petitioner was issued to Jerry Valeroso by virtue of a Memorandum Receipt.

Issues:
1. Was there a valid warrantless search in this case?

Ruling:
1. NO. Firstly, the case does not fall under search incidental to a lawful arrest. The purpose of this
principle, as enunciated in Section 13 of Rule 126 of the Rules of Court is to protect the arresting officer from danger, by
searching for and seizing any weapon in the immediate vicinity that the arrested person may use. In the case at bar, no
such danger is present. From the facts, it can be averred that the arresting officers served the warrant without any
resistance from Valeroso. Moreover, the cabinet that was subsequently searched was locked, and as such is no longer
considered an area “within the immediate control” of the accused. The arresting officers would have been justified in
searching the person of Valeroso, as well as the tables or drawers in front of him, for any concealed weapon that might
be used against the former. But under the circumstances obtaining, there was no comparable justification to search
through all the desk drawers and cabinets or the other closed or concealed areas in that room itself.
Valeroso v. People
[EX POST FACTO LAW]

Facts:
By virtue of a duly issued warrant of arrest, petitioner Valeroso was arrested in July of 1996 for a count
of kidnapping for ransom. When he was searched incident to the arrest, a firearm with live ammunition was found
tucked by his waist. The firearm was found to be licensed, not to Valeroso, but to another person. Petitioner was then
charged with illegal possession of firearm and ammunition under Presidential Decree (P.D.) No. 1866, which was the
governing law during the commission of the offense. Under this law, manufacture, sale, acquisition, disposition, and
possession of firearms are punishable with reclusion temporal in its maximum to reclusion perpetua. However, during
the pendency of the case with the trial court, R.A. 8294 amended PD 1866. The former imposed for the same offense a
penalty of prision correccional in its maximum period and a fine of fifteen thousand pesos.

Issues:
1. May the new law be retroactively applied and effectively lessen Valeroso’s penalty?

Ruling:
1. YES. As a general rule, penal laws should not have retroactive application, lest they acquire the
character of an ex post facto law. An exception to this rule, however, is when the law is advantageous to the accused.
According to Mr. Chief Justice Araullo, this is “not as a right” of the offender, “but founded on the very principles on
which the right of the State to punish and the commination of the penalty are based, and regards it not as an exception
based on political considerations, but as a rule founded on principles of strict justice.” Although an additional fine of
P15,000.00 is imposed by R.A. No. 8294, the same is still advantageous to the accused, considering that the
imprisonment is lowered to prision correccional in its maximum period
People v. Bolasa
[SEARCHES AND SEIZURES]

Facts:
An anonymous caller tipped off some police officers that a man and a woman were repacking prohibited
drugs at a certain house in Sta. Brigida St., Karuhatan, Valenzuela, Metro Manila. The officers immediately proceeded
to the house of the suspects and upon reaching the house, peeped inside through a small window, and confirmed that
there were indeed a couple packing drugs. They arrested the accused and filed a case against them. Respondents’
defense hinges on illegal search.

Issues:
1. Was there a valid warrantless search in this case?

Ruling:
1. NO. The marijuana were not seized in plain view or inadvertently discovered. There was no valid
intrusion and the accused were illegally arrested. The police officers intentionally peeped first through the window
before they saw and ascertained the activities of accused inside the room. Further, the Court contended that the
apprehending officers should have conducted first a surveillance considering that the identities and address of the
suspected culprits were already ascertained. After conducting the surveillance and determining the existence of
probable cause for arresting accused, they should have secured a search warrant prior to effecting a valid arrest and
seizure. The Court stated that the arrest being illegal ab initio, the accompanying search was likewise illegal.
Chavez v. Gonzales
[FREEDOM OF EXPRESSION]

Facts:
On June 11, 2005, after the issue of the “Hello Garci” scandal first came into light, the National
Telecommunications Commission issued a press release warning radio and television stations that airing the
aforementioned recordings is a cause for the suspension, revocation, and/or cancellation of the license or
authorizations issued them.

Issues:
1. May the NTC validly enforce restraint on the right of radio and television stations to air the Garci
tapes?

Ruling:
1. NO. The mandate of Section 4 of Article III of the Bill of Rights is that the freedom of expression and of
the press, or the right of the people peaceably to assemble cannot be abridged. In essence, it prohibits content-based
prior restraint or censorship of the freedom of expression. What the Constitution allows is merely content-neutral
prohibition, as when a permit restrains or regulates the time, place or manner of the expression or assembly, especially
when conducted in a public place or through busy streets. The NTC’s act of prohibiting press freedom based on content
considerations therefore, is in violation of the Bill of Rights.
Newsounds Broadacasting v. Dy
[FREEDOM OF EXPRESSION]

Facts:
Petitioners are the operators of Bombo Radyo DZNC Cauayan, an AM radio broadcast station based in
Cauayan City, Isabela. In 2002, petitioners applied for the renewal of their Mayor’s Permit, but city zoning administrator-
designate Bagnos Maximo refused to issue zoning clearance on the grounds that Newsounds was not able to submit
conversion papers showing that the agricultural land that houses their station has been converted to commercial land.
After repeated petitions were denied, Newsounds filed a case, claiming that the city government of Cauayan was
discriminating against it because Bombo Radyo DZNC has been fervently calling officials out on their corruption.

Issues:
1. Is there violation of the freedom of expression in this case?

Ruling:
1. YES. Respondents closure of petitioner’s radio stations is clearly tainted with ill motvies. Petitioners
have been aggressive in exposing the widespread election irregularities in Isabela that appear to have favored
respondent Dy and his political dynasty. Such statement manifests and confirms that respondent’s denial of the renewal
applications on the ground that property is commercial and merely a pretext, and their real agenda is to remove
petitioners from Cauayan City and suppress the latter’s voice. This is a blatant violation of constitutional right to press
freedom.
Borjal v. Court of Appeals
[FREEDOM OF EXPRESSION]

Facts:
Borjal published in his editorial column for a popular newspaper certain anomalous activities of an
“organizer of a conference”. Francisco Wenceslao, believing himself to be the target of these offensive and derogatory
remarks, filed a civil action for damages on the basis of libel. Wenceslao however was never named in any of the
articles nor was the conference he was organizing. The lower court and the CA agreed that the remarks have
sufficiently established the identity of private respondent Wenceslao as the aforementioned “organizer of the
conference”.

Issues:
1. Can petitioner hold the defense that his articles constitutes unactionable privileged communication?

Ruling:
1. YES. While the act of petitioner does not fall within the actionable exceptions under Article 354 of the
Revised Penal Code, as a newspaper article is neither a private communication nor a fair and true report without
comments or remarks, fair commentaries on matters of public interest are privileged and constitute a valid defense in an
action for libel or slander. The doctrine of fair comment means that while in general every discreditable imputation
publicly made is deemed false, because every man is presumed innocent until his guilt is judicially proved, and every
false imputation is deemed malicious, nevertheless, when the discreditable imputation is directed against a public
person in his public capacity, it is not necessarily actionable. The freedom of expression and the press is constitutionally
protected, because the press is the fidei defensor, or defender of the faith, of the people in a democratic society.
Tulfo v. People
[FREEDOM OF EXPRESSION]

Facts:
Atty. Carlos So is suing Tulfo for libel, after having written and published in the Remate tabloid,
derogatory remarks against him. The article personally named So, and even went so far as to ridicule and throw curse
words at him, with the intent of exposing him to public hatred and contempt.

Issues:
1. Can petitioner hold the defense that his articles constitutes fair and true reports as contemplated in the
exemptions under Article 354 of the Revised Penal Code?

Ruling:
1. NO. It is clear upon reading the tabloid articles that the remarks are little more than baseless
accusations., backed up by the word of an unnamed source. Good faith is lacking, as Tulfo failed to substantiate or
even attempt to verify his story before publication. Tulfo goes even further to attack the character of the subject, Atty.
So, even calling him a disgrace to his religion and the legal profession. As none of the elements of the second
paragraph of Art. 354 of the RPC is present in Tulfo’s articles, it cannot thus be argued that they are qualified privileged
communications under the RPC. Breaking down the provision further, looking at the terms "fair" and "true," Tulfo’s
articles do not meet the standard. "Fair" is defined as "having the qualities of impartiality and honesty." "True" is defined
as "conformable to fact; correct; exact; actual; genuine; honest." Tulfo failed to satisfy these requirements, as he did not
do research before making his allegations, and it has been shown that these allegations were baseless.
Kilusang Mayo Uno v. Ermita
[FREEDOM OF EXPRESSION]

Facts:
Petitioners herein assail the constitutionality of R.A. 9372, or the Anti-Terrorism Law for, among other
reasons, being intrinsically vague and impermissibly broad, particularly in defining the crime of terrorism. They claim
that terms like “widespread and extraordinary fear and panic among the populace and coerce the government to give in
to an unlawful demand” are nebulous, leaving law enforcement agencies with no standard to measure the prohibited
acts.

Issues:
1. May the petitioners institute a facial challenge against RA 9372 on the grounds of the void-for-
vagueness and overbreadth doctrine?

Ruling:
1. NO. In the outset, RA 9372 cannot be facially challenged, because it regulates conduct, and not
expression. A facial challenge is exclusively available only to what is known in American law as First Amendment rights,
that is: the rights to speech, expression, the press, assembly and petition for redress of grievances, and the non-
establishment of religion. The allowance of a facial challenge in free speech cases is justified by the aim to avert the
chilling effect on protected speech, the exercise of which should not at all times be abridged. As reflected earlier, this
rationale is inapplicable to plain penal statutes that generally bear an in terrorem effect in deterring socially harmful
conduct. In fact, the legislature may even forbid and penalize acts formerly considered innocent and lawful, so long as it
refrains from diminishing or dissuading the exercise of constitutionally protected rights.

Important note:
The Supreme Court in this case enunciated the difference between the void-for-vagueness and overbreadth doctrines,
to wit:
A statute or act suffers from the defect of vagueness when it lacks comprehensible standards that men of common
intelligence must necessarily guess at its meaning and differ as to its application. It is repugnant to the Constitution in
two respects: (1) it violates due process for failure to accord persons, especially the parties targeted by it, fair notice of
the conduct to avoid; and (2) it leaves law enforcers unbridled discretion in carrying out its provisions and becomes an
arbitrary flexing of the Government muscle. T he overbreadth doctrine, meanwhile, decrees that a governmental
purpose to control or prevent activities constitutionally subject to state regulations may not be achieved by means which
sweep unnecessarily broadly and thereby invade the area of protected freedoms.
As distinguished from the vagueness doctrine, the overbreadth doctrine assumes that individuals will understand what
Ebralinag v. Division Superintendent of Schools
[FREEDOM OF RELIGION]

Facts:
As a consequence of Jehova’s Witness children enrolled in various public and private schools refusing to
sing the national anthem, salute the flag, and recite the patriotic pledge, DECS Division Superintendent of Schools
Susan B. Cabahug of the Cebu Division issued a memorandum directing District Supervisors, High School Principals,
and Heads of Private Educational Institutions to deprive the aforementioned students and pupils from the benefit of
public education, if they do not participate on daily flag ceremony.

Issues:
1. Is this memorandum in violation of the children’s right to exercise their religious beliefs?

Ruling:
1. YES. It is illogical to think that exempting Jehova’s Witnesses children from participating in the daily
flag ceremony in public schools will have such an adverse effect in the public welfare so as to create an immediate and
grave danger. In stark contrast, forcing the members of a small religious group, through the iron hand of the law, to
participate in a ceremony that violates their religious beliefs will hardly be conducive to love of country or respect for its
duly constituted authorities.
Islamic Da’wah Council v. Executive Secretary
[FREEDOM OF RELIGION]

Facts:
Petitioner is to declare the nullity of Executive Order No. 46, which created the Philippine Halal
Certification Scheme and designating respondent Office of Muslim Affairs to oversee the registration of halal certified
food products in the country. IDCP claims that it has already been extending the same service for the benefit of the
Muslim community before the enactment of the EO.

Issues:
1. Does EO 46 violate freedom of religion?

Ruling:
1. YES. It is unconstitutional for the government to formulate policies and guidelines on the halal
certification scheme because said scheme is a function only religious organizations, entity or scholars can lawfully and
validly perform for the Muslims. According to petitioner, a food product becomes halal only after the performance of
Islamic religious ritual and prayer. Thus, only practicing Muslims are qualified to slaughter animals for food. A
government agency like herein respondent OMA cannot therefore perform a religious function like certifying qualified
food products as halal.
Estrada v. Escritor
[FREEDOM OF RELIGION]

Facts:
Soledad Escritor, a court interpreter, is under fire for living with a man not her husband. They allegedly
have a child of eighteen years. Petitioner Alejandro Estrada claims that this fact constitutes immorality unbecoming of a
public employee and tarnishes the image of the courts, and so he asks for the termination of respondent. Escritor, in
reply, testified that she was already a widow when she joined the judiciary in 1999. Nevertheless, according to her, her
faith as a member of the Jehova’s Witness religious sect does not make her act immoral. In her congregation,
cohabiting with a man without the benefit of marriage is not looked down upon.

Issues:
1. Can Escritor be spared from termination on the ground of religious freedom?

Ruling:
1. YES. The doctrine of benevolent neutrality, which our judiciary has adopted in interpreting the religious
freedom guaranteed by the Constitution, recognizes that the government must pursue its secular goals and interests
while striving to uphold religious liberty to the greatest extent possible within the most flexible of constitutional limits.
Therefore, morality on the ground of religious belief is allowed, provided it does not offend compelling state interests. In
this test, it is not enough to simply show a rational relationship of the substantial infringement to the religious right and a
colorable state interest. Instead, only the gravest abuses, endangering paramount interests, give occasion to
permissible limitation. Applying the test in the case at bar, there is no compelling state interest to be protected that will
justify respondent’s termination.
Victoriano v. Elizalde Rope Workers’ Union
[FREEDOM OF RELIGION / FREEDOM OF ASSOCIATION / NON-IMPAIRMENT CLAUSE]

Facts:
Benjamin Victoriano, a member of the Iglesia ni Cristo, was employed in the Elizalde Rope Factory, Inc.
since 1958. He was a member of the Elizalde Rope Workers’ Union, which had a closed shop agreement with the
factory, which required that membership to the union is a requirement for the employment of all permanent employees
of the company. Victoriano’s religious beliefs however, precluded him from joining in any union or organization. When
he asked to resign from the union, the latter called for his termination.

Issues:
1. Can Victoriano be exempt from the closed shop agreement on the basis of his religion?
2. Can the state in this case validly encroach upon the autonomy of two contracting parties?

Ruling:
1. YES. The right to religion prevails over contractual or legal rights. As such, an INC member may
refuse to join a labor union and despite the fact that there is a close shop agreement in the factory where he was
employed, his employment could not be validly terminated for his non-membership in the majority therein. Moreover, the
freedom to join an association includes the freedom not to associate, so the union cannot compel Victoriano to become
a member.
2. YES. In many a case, the Supreme Court has declared that the prohibition against impairing the
obligation of contracts has no application to statutes relating to public subjects within the domain of the legislative
powers of the state involving public welfare. No unchanging yardstick in determining whether legislation
unconstitutionally impairs contract obligations has been defined, but as a general rule, the promotion of the common
good, within the virtue of valid police power, may call for the setting aside of the non-impairment clause.
Mañosca v. Court of Appeals
[FREEDOM OF RELIGION]

Facts:
Petitioners inherited a piece of land located at Taguig. When the parcel of land was ascertained by the
NHI to have been the birth site of Felix Y. Manalo, the founder of Iglesia Ni Cristo, it passed a resolution granting
National Landmark Status to it. Petitioners moved to dismiss the complaint on the main thesis that the intended
expropriation was not for a public purpose and, incidentally, that the act would constitute an application of public funds,
directly or indirectly, for the use, benefit, or support of Iglesia ni Cristo, a religious entity, contrary to the provision of
Section 29(2), Article VI, of the 1987 Constitution.

Issues:
1. Is the contention valid?

Ruling:
1. NO. The fact that the greater benefit from the appropriation may be derived by members of the INC is
such an incidental advantage, and as such it is merely secondary in nature. It cannot be inferred that this is in violation
of the non-establishment clause, as Felix Manalo is nevertheless a major contributor to the culture and history of the
Philippines. Moreover, public use in terms of eminent domain should not be restricted only to traditional uses.
ANG LADLAD v. COMELEC
[FREEDOM OF RELIGION]

Facts:
ANG LADLAD is a party list group that represents members of the lesbian, gay, bisexual, and
transgender community. COMELEC is looking for its disqualification for party list status on the grounds of morality.

Issues:
1. Is the proposed disqualification correct?

Ruling:
1. NO. The fact that the COMELEC relied on religious justification to deny a party list representing the
lesbian, gay, bisexual and transgender (LGBT) community is inconsistent with the principle enshrined in Article III,
Section 5 of the Constitution, which provides: “No law shall be made respecting an establishment of religion or
prohibiting the free exercise thereof.” COMELEC also failed to explain what societal ills are sought to be prevented, or
why special protection is needed against the establishment of such a group.
Kuwait Airways v. Philippine Airlines
[NON-IMPAIRMENT CLAUSE]

Facts:
In 1981, Kuwait Airways and Philippine Airlines (acting then as a government-owned and controlled
corporation) entered into a Commercial Agreement wherein some of the operations and exercise of certain freedom
traffic rights between the two nations were subject to royalty payments. The contract persisted even as PAL was
privatized in 1992. On May 1995, delegates of the governments of Kuwait and the Philippines, including officials of PAL,
signed a Confidential Memorandum of Understanding (CMU). The new contract stipulated that the exercise of third or
fourth freedom traffic rights shall no longer be subject to royalty payment or commercial arrangements. In relation to
this, Kuwait Airways sent a letter to PAL stating that it will terminate the original agreement on April 12, 1995. PAL
however, replied that pursuant to the original agreement, termination may only be made on the last day of any traffic
period, which means it could only cease to take effect on October 31, 1995. It filed a petition for the fulfillment of Kuwait
Airways’ obligation of payment during the period of April to October 1995.

Issues:
1. May a private corporation (PAL) be subjected to a contract signed by the executive department (CMU)
which impairs its obligation?

Ruling:
1. NO. One of the most essential characteristics of contracts is its obligatory force, which means that it
binds contracting parties as the source of vested rights which may not be impaired. In the immediate case, PAL was
already a private corporation when the CMU was signed, therefore it was no longer controlled by the government. In
order to validly circumvent the non-impairment clause, the government should first observe due process and seek non-
invasive remedies provided for under civil law. For one, the government could have called for the judicial annulment or
reformation of the contract to effect the termination of the original commercial agreement instead.

Important note:
See Arts. 1306, 1308, and 1317 of the NCC
Secretary of Justice v. Judge Lantion
[RIGHTS OF THE ACCUSED – DUE PROCESS IN CRIMINAL PROCEEDINGS]

Facts:
On June 1999, the government, through the Department of Justice, received a request from the United
States of America, through the Department of Foreign Affairs, calling for the extradition of Mark Jimenez. This is
pursuant to PD No. 1609 prescribing the procedure for the extradition of persons who have committed a crime in a
foreign country. Jimenez sought copies of the extradition requests and asked that he be given ample time to comment
on said requests. Herein petitioner denied the same.

Issues:
1. Should treaty stipulations prevail over an individual’s right to due process?

Ruling:
1. NO. It is a known principle of statutory construction that when a municipal law and an international law
is in conflict, the local law must prevail. This rule is no less applicable given that it is the Constitution itself that provides
that the accused in a criminal proceeding shall be informed of the nature and cause of the accusation against him. The
fact that international law has been made part of the law of the land does not pertain to or imply the primacy of
international law over municipal law in the municipal sphere. In states where the constitution is the highest law of the
land, both statutes and treaties may be invalidated if they are in conflict with the constitution.
U.S. v. Judge Puruganan
[RIGHTS OF THE ACCUSED – BAIL]

Facts:
The Supreme Court, after ruling in favor of Mark Jimenez in Secretary of Justice v. Lantion, reversed its
order to furnish Mr. Jimenez copies of the extradition request and its supporting papers. Subsequently, the US
government filed a petition for extradition with a prayer calling for the arrest of Mr. Jimenez. After the hearing, the
subject submitted his memorandum to the extradition court, seeking an alternative prayer that in case a warrant of
arrest should issue, he be allowed to post bail in the amount of ₱10,000.

Issues:
1. Should Jimenez be allowed to post bail?

Ruling:
1. NO. The constitutional provision on bail on Article III, Section 13 of the Constitution, as well
as Section 4 of Rule 114 of the Rules of Court, applies only when a person has been arrested and detained for violation
of Philippine criminal laws. It does not apply to extradition proceedings, because extradition courts do not render
judgments of conviction or acquittal. Extradition proceedings are separate and distinct from the trial for the offenses for
which he is charged. He should apply for bail before the courts trying the criminal cases against him, not before the
extradition court.
On the other hand, bail is not a matter of right in extradition cases. Instead, it is subject to judicial
discretion in the context of the peculiar facts of the case. Bail in extradition cases may only be applied for when there is
a clear and convincing showing:
1)That, once granted, the applicant will not be a flight risk or a danger to the community, and that
2)There exists special, humanitarian, and compelling circumstances including, as a matter of reciprocity, those cited by
the highest court in the requesting state when it grants provisional liberty in extradition cases therein
Government of Hongkong Special Administrative Region v. Judge Olalia
[RIGHTS OF THE ACCUSED – BAIL]

Facts:
Juan Antonio Muñoz was charged before the Hongkong court with three counts of the offense of
“accepting an advantage as an agent”. As such, herein petitioner filed a petition for the extradition of the private
respondent. Moreover, private respondent filed a petition for bail, which respondent judge approved conditionally.
Muñoz was arrested and detained in 1996.

Issues:
1. Should Muñoz be allowed to post bail?

Ruling:
1. NO. According to Justice Reynato Puno in his separate opinion in the Puruganan case, the standard of
proof required in granting or denying bail can neither be the proof beyond reasonable doubt in criminal cases nor the
standard of proof of preponderance of evidence in civil cases. While administrative in character, the standard of
substantial evidence cannot likewise apply, given that the object of an extradition proceeding is to prevent the
prospective extraditee from fleeing the jurisdiction of the state. Wherefore, he proposed a new standard exclusive for
bail petitions in extradition cases, which he termed “clear and convincing evidence”. In the aforementioned case, there
is no showing that the private respondent presented clear and convincing evidence that he is not a flight risk. Absent
such proof, he cannot be granted bail.
Re: Request for Live Radio-TV Coverage of the Plunder Trial against Former
President Joseph E. Estrada
[RIGHT TO INFORMATION / RIGHTS OF THE ACCUSED – FAIR TRIAL]

Facts:
Members of the press claim that the trial on the plunder cases of former President Estrada before the
Sandiganbayan should be broadcast live, in line with the constitutionally-protected rights of the press and to information
on matters of public concern. The former President claims that doing so will violate his right to a speedy and fair trial, as
well as trigger massive demonstrations and “expert opinions” which could distract or sway the court one way or the
other.

Issues:
1. Should the trial be aired?

Ruling:
1. YES. The Supreme Court granted the audio-visual coverage of the trial, but only as a matter of
documentation. Only later will the copies be available for public viewing. This way, concerns about the regularity and
fairness of the trial -- which, it may be assumed, is the concern of those opposed to, as much as of those in favor of,
televised trials - will be addressed since the tapes will not be released for public showing until after the decision of the
cases by the Sandiganbayan. By delaying the release of the tapes, much of the problem posed by real time TV and
radio broadcast will be avoided. The Court also turned down the defense of violation of privacy, for “A limited intrusion
into a person's privacy has long been regarded as permissible where that person is a public figure and the information
sought to be elicited from him or to be published about him constitute matters of a public character.”
People v. Escaño
[RIGHTS OF THE ACCUSED – PRESUMPTION OF INNOCENCE]

Facts:
Escaño, Usana, and Lopez were charged with violation of various special penal laws, and the cases
were consolidated and tried jointly. After being convicted, the three filed separate appeals, but Escaño withdrew his
appeal. When Usana and Lopez were subsequently acquitted by the Supreme Court, Escaño claimed that the judgment
should likewise apply to him, notwithstanding his withdrawal.

Issues:
1. Should Escaño be acquitted?

Ruling:
1. YES. Section 11(a), Rule 122 of the Rules on Criminal Procedure states that an appeal taken by one
or more of several accused shall not affect those who did not appeal, except insofar as the judgment of the appellate
court is favorable and applicable to the latter. The consolidated cases being of the same nature, and the Supreme Court
having found that not all the elements of the crime are present, the acquittal of Usana and Lopez based on reasonable
doubt should likewise benefit Escaño.
Pharmaceutical and Health Care Association v. Secretary of Health
[DUE PROCESS]

Facts:
On October 28, 1986, Executive Order No. 51 (Milk Code) was issued by President Corazon Aquino by
virtue of the legislative powers granted to the president under the Freedom Constitution. Petitioners assail its
constitutionality on various grounds, among them are the provisions (Sections 4(f), 4(i), 5(w), 11, 22, 32, 46, 52) that
suppress the trade of milk. According to petitioner, this constitutes a violation of the due process clause of the
Constitution, as it invariably prohibits the trade of breastmilk substitutes.

Issues:
1. Was there a violation of due process in this case?

Ruling:
1. NO. As a general rule, trade may be subjected to some form of regulation when done for the common
good. Indeed, public interest must prevail over business interests. In this case, petitioner failed to show that the
proscription of milk manufacturers participation in any policymaking body (Section 4(i)), classes and seminars for
women and children (Section 22); the giving of assistance, support and logistics or training (Section 32); and the giving
of donations (Section 52) would unreasonably hamper the trade of breastmilk substitutes. Petitioner has not established
that the proscribed activities are indispensable to the trade of breastmilk substitutes. Petitioner failed to demonstrate
that the aforementioned provisions of the RIRR (the Milk Code’s Implementing Rules and Regulations) are
unreasonable and oppressive for being in restraint of trade.
Bayan v. Ermita
[FREEDOM OF PEACEFUL ASSEMBLY]

Facts:
Petitioners are attacking the validity of Batas Pambansa Blg. 880, otherwise known as the Public
Assembly Act of 1985. They contend that the aforementioned law violates the Constitutional guarantee of the freedom
to peaceably assemble, because it compels rallyists to obtain a permit so that they can stage a public assembly,
regardless of the presence of a clear and present danger. It also curtails the choice of venue. Also, the phrase
“maximum tolerance” shows that the law applies to assemblies against the government because they are being
tolerated. As a content-based legislation, it cannot pass the strict scrutiny test.

Issues:
1. Does BP 880 violate Section 4, Article III of the 1987 Constitution?

Ruling:
1. NO. As a general rule, the freedom of peaceful assembly enjoys dominance in the sphere of
constitutional protection, along with the freedom of speech, expression, of the press, and religion. These rights
represent the very basis of a functional democratic polity, without which all the other rights would be meaningless and
unprotected. However, it must be remembered that the right, while sacrosanct, is not absolute. It may be regulated that
it shall not be injurious to the equal enjoyment of others having equal rights, nor injurious to the rights of the community
or society. B.P. No 880 is not an absolute ban of public assemblies but a restriction that simply regulates the time, place
and manner of the assemblies.
Chavez v. Public Estates Authority and Amari Coastal Bay Development
[RIGHT TO INFORMATION]

Facts:
Petitioner Frank Chavez is assailing the validity of the Joint Venture Agreement between the Public
Estates Authority, a government owned and controlled corporation, and Amari Coastal Bay Development, a private
corporation, regarding the development and reclamation of the “Freedom Islands”, a track of land expropriated under
eminent domain for public use. In filing his petition for prohibition with application for the issuance of a temporary
restraining order and preliminary injunction, he asked that PEA publicly disclose the terms of any renegotiations of the
JVA.

Issues:
1. Does the constitutional right to information on matters of public concern include official information on
ongoing negotiations before a final agreement?

Ruling:
1. NO. The right to information, as stated in Section 7, Article III of the 1987 Constitution, and reinforced
by Section 28, Article II of the same charter, is aimed towards the promotion of transparency in policy-making and in the
operations of the government, as well as provide the people sufficient information to exercise effectively other
constitutional rights. These twin provisions are essential to the exercise of freedom of expression. However, distinction
between the constitutional guarantee of the right to information and the information incidental to the conduct of a public
bidding must first be made. The latter (the size, location, technical description and nature of the property being disposed
of, the terms and conditions of the disposition, the parties qualified to bid, the minimum price and similar information)
must be disclosed without demand, because the Government Auditing Code requires a public bidding. Information,
however, on on-going evaluation or review of bids or proposals being undertaken by the bidding or review committee is
not immediately accessible under the right to information. While the evaluation or review is still on-going, there are no
"official acts, transactions, or decisions" on the bids or proposals. However, once the committee makes its official
recommendation, there arises a "definite proposition" on the part of the government. From this moment, the public's
right to information attaches, and any citizen can access all the non-proprietary information leading to such definite
proposition.
Hilado v. Judge Reyes
[RIGHT TO INFORMATION]

Facts:
The petitioners in this case are alleged creditors of the estate of Roberto Benedicto. In the midst of the
intestate proceedings for the aforementioned estate before Judge Amor Reyes of the Manila RTC, petitioners filed
various requests for the access to records of the case and to secure certified true copies thereof, all of which were
denied by herein public respondent.

Issues:
1. When may court records fall under the constitutional provision on the right to information on matters of
public concern?

Ruling:
1. As a general rule, judicial records may or may not be accessed under the right to information clause,
depending on a case-to-case basis. The term judicial record or court record does not only refer to the orders, judgment
or verdict of the courts. It comprises the official collection of all papers, exhibits and pleadings filed by the parties, all
processes issued and returns made thereon, appearances, and word-for-word testimony which took place during the
trial.
Decisions and opinions of a court are of course matters of public concern or interest for these are the
authorized expositions and interpretations of the laws, binding upon all citizens, of which every citizen is charged with
knowledge. Unlike court orders and decisions, however, pleadings and other documents filed by parties to a case need
not be matters of public concern or interest. In thus determining which part or all of the records of a case may be
accessed to, the purpose for which the parties filed them is to be considered.
In short, as long as any party, counsel, or person has a legitimate reason to have a copy of court records
and pays court fees, a court may not deny access to such records. Of course, this rule is subject to certain
precautionary measures to prevent tampering or alteration of the documents.

Important note:
SC ruled in favor of the creditors insofar as it allowed them access to the records of the case, subject to aforementioned
precautionary measures.
Miriam College Foundation, Inc. v. Court of Appeals
[FREEDOM OF EXPRESSION]

Facts:
Obscene, vulgar, indecent, gross, sexually explicit, injurious to young readers, and devoid of all moral
values. This was how some members of the Miriam College community allegedly described the contents of the
September-October 1994 issue (Vol. 41, No. 14) of Miriam Colleges school paper (Chi-Rho), and magazine (Ang
Magasing Pampanitikan ng Chi-Rho). Consequently, the College Board asked the students to submit a written
statement in answer to the charges filed. The challenge went unanswered. When a case was finally filed against them,
the students invoked their right to free speech and to peaceably assemble.

Issues:
1. Does the aforementioned acts of obscenity violate the freedom of expression guaranty?

Ruling:
1. NO. Justice Fortas, in Tinker v. Des Moines Community School, stated that students do not “shed their
constitutional rights to freedom of speech or expression at the schoolhouse gate.” While, therefore, the authority of
educational institutions over the conduct of students must be recognized, it cannot go so far as to be violative of
constitutional safeguards. Nevertheless, the right of the students to free speech in school premises is not absolute.
Expression must always be applied in light of the special characteristics of the school environment.

Important note:
SC upheld the school’s power to investigate breaches of campus conduct and enforce rules and regulations in the
maintenance of a safe and orderly educational environment conducive to learning.
Isagani Cruz v. Secretary of Environment and Natural Resources
[EMINENT DOMAIN]

Facts:
Former Justice Isagani Cruz, a noted constitutionalist, assailed the validity of the Republic Act No. 8371
or the Indigenous People’s Rights Act (IPRA Law) on the ground that the law amount to an unlawful deprivation of the
State’s ownership over lands of the public domain as well as minerals and other natural resources therein, in violation of
the regalian doctrine embodied in Section 2, Article XII of the Constitution.

Issues:
1. Does RA 8371 deprive the government of publicly-owned lands?

Ruling:
1. NO. In the outset, ancestral lands and domain do not form part of the lands under public domain.
Instead, they are lands belonging to indigenous people by native title, a concept of private land ownership that has
outlived the modern standards of ownership, and exists irrespective of any grant from the State. Moreover, the IPRA
merely grants to indigenous people, as owners and occupants of their lands, small-scale utilization of the natural
resources found therein, with respect to their large-scale development and exploitation.
Social Weather Stations, Inc. v. COMELEC
[FREEDOM OF EXPRESSION]

Facts:
The SWS, in the service of conducting surveys in various fields, among which is the political arena, is
looking to enjoin the COMELEC from enforcing R.A. 9006, or the Fair Election Act. It assails, among others, that the
provision that survey results of national concern shall not be published fifteen days before elections, and seven days in
the case of surveys affecting local candidates is in violation of the constitutional safeguard on the right to free
expression and of the press.

Issues:
1. Does the Fair Election Act violate the freedom of expression?
Ruling:
1. YES. Section 5(4) of the Fair Election Act is invalid because it imposes prior restraint on the freedom
of expression. As if to aggravate its invalidity, the assailed provision is also a direct and total suppression of a category
of expression, even if such suppression is only for a limited period. Moreover, the governmental interest sought to be
protected can be achieved by means other than complete suppression of the freedom of expression.
Soriano v. Laguardia
[FREEDOM OF EXPRESSION]

Facts:
Petitioner, as host of the program Ang Dating Daan, made certain obscene remarks against Iglesia ni
Cristo. Two days after, members of the INC filed separate but identical complaints against petitioner. MTRCB thereafter
suspended him for three months.

Issues:
1. Is the act of suspending the petitioner valid and constitutional?
Ruling:
1. YES. Under the circumstances obtaining in this case, and considering the adverse effect of petitioner's
utterances on the viewers‟ fundamental rights as well as petitioner's clear violation of his duty as a public trustee, the
MTRCB properly suspended him from appearing in Ang Dating Daan. Moreover, he cannot properly assert that his
suspension was an undue curtailment of his right to free speech either as prior restraint or subsequent punishment.
Iglesia ni Cristo v. Court of Appeals
[FREEDOM OF EXPRESSION / FREEDOM OF RELIGION]

Facts:
Private respondent Board of Review for Moving Pictures and Television (now the MTRCB), issued an X
rating (not for public viewing) on the program “Ang Iglesia ni Cristo” of the religious group of the same name, for
allegedly “offending and constituting an attack against other religions which is expressly prohibited by law.” This is due
to the inclusion of controversial biblical interpretations and its attacks against contrary religious beliefs. INC contends
that this is in violation of their freedom of expression and religion.

Issues:
1. Is the rating valid and constitutional?
Ruling:
1. NO. Deeply ensconced in our fundamental law is its hostility against all prior restraints on speech,
including religious speech. Hence, any act that restrains speech is hobbled by the presumption of invalidity and should
be greeted with furrowed brows. It is the burden of the respondent Board to overthrow this presumption. If it fails to
discharge this burden, its act of censorship will be struck down. In the case at bar, the so-called “attacks” are nothing
but mere criticisms of some of the more deeply held dogmas and tenets of other religions. This does not constitute a
clear and present threat by which the right to religious freedom may be suspended. The respondent Board may
disagree with the criticisms of other religions by petitioner but that gives it no excuse to interdict such criticisms,
however, unclean they may be. Under our constitutional scheme, it is not the task of the State to favor any religion by
protecting it against an attack by another religion.
Hilado v. Judge Reyes
[RIGHT TO INFORMATION]

Facts:
Petitioner is a creditor of the estate of Roberto Benedicto, and he, along with other creditors, requested
Judge Reyes to grant him access to court records of the intestate proceedings while the same was still pending in the
RTC of Bacolod City. The respondent judge denied the motion, claiming that the complainants were not considered
parties-in-interest since their claims remain contingent on the outcome of pending cases. Moreover, she claimed that
the complainants’ interest is more personal than public concern.

Issues:
1.Was Judge Reyes correct in denying access to court documents?
Ruling:
1. NO. The Supreme Court decided that the respondent judge arbitrarily denied the complainants’ access
to case records. Admittedly, the complainants could not demand that they be furnished with the courts orders and the
pleadings filed by the parties, in as much as the respondent Judge had already ruled that they were not parties-in-
interest. However, the Court finds that the respondent Judge should not have prohibited the complainants from going
over the records of the case and securing copies of pertinent orders and pleadings. Courts in the United States have
recognized the general right to inspect and copy public records and documents, including judicial records and
documents. In our jurisdiction, the right is enshrined in Section 7, Article III of the Constitution

Important note:
Note that this is a different case than the one of the same name (71), albeit petitioner and private respondent are
similar. Read the full text for more details on the disparity.
AKBAYAN v. Aquino
[RIGHT TO INFORMATION]

Facts:
Through a petition for mandamus and prohibition, petitioners request respondents to furnish copies of the
full text of the Japan-Philippines Economic Partnership Agreement (JPEPA), as well as all documents pertaining to the
approval of the treaty, including offers made by each contracting party. Petitioner emphasize that the refusal of the
government to disclose the said agreement violates there right to information on matters of public concern and of public
interest.

Issues:
1. Are documents relating to the approval of a treaty exempt from the general rules on transparency and
full disclosure?

Ruling:
1. NO. As a general rule, the final text of a treaty may not be kept perpetually confidential. However,
there should be ample time and opportunity for discussion before a treaty may be approved, and as such, the offers
exchanged by the parties during this negotiation period continue to be privileged even after the treaty has been
published. Disclosing the content of the cards on the negotiation table could betray the trust of foreign states in the
confidentiality of Philippine diplomatic relations. Opening the offers would discourage future Philippine representatives
from expressing their full views during negotiation.
National Power Corporation v. Maria Mendoza San Pedro
[EMINENT DOMAIN]

Facts:
For the construction of its San Manuel-San Jose 500 KV Transmission Line and Tower No. SMJ-389,
NPC negotiated with Maria Mendoza San Pedro, then represented by her son, Vicente, for an easement of right of way
over her property, a track of land which was partly agricultural and partly residential. The payment voucher for the
residential portion of the lot valued at P6,000,000.00 (at P600.00 per square meter) was then processed. However, the
NPC Board of Directors approved Board Resolution No. 97-2468 stating that it would pay only P230.00 per sq m for the
residential portion and P89.00 per sq m for the agricultural portion. Petitioner on the other hand, likewise argued that the
property involved was actually and principally used as agricultural, though declared as agricultural/residential lots;
hence, only the easement fee of right of way should be paid, as the principal purpose for which the lot was devoted
would not be impaired by the construction of transmission lines.

Issues:
1. Should there be payment of just compensation even when the property taken is merely a right-of-way
easement?

Ruling:
1. YES. Expropriation is not limited to the acquisition of real property with a corresponding transfer of title
or possession. The Supreme Court in NPC v. Aguirre-Paderanga ruled that a right-of-way easement resulting in a
restriction or limitation on property rights over land falls within the ambit of the term “expropriation”. In the case at bar,
the easement of right-of-way is definitely a taking under the power of eminent domain.

Important note:
There are a few more (rather complicated) discussions on the computation of just compensation and eminent domain in
general mentioned in this case. Read the full case for more information.
Air Transportation Office v. Apolonio Gopuco
[EMINENT DOMAIN]

Facts:
Respondent is the owner of a 995-square meter land located in the vicinity of the Lahug Airport in Cebu
City. When private land is expropriated for a particular public use, and that particular public use is abandoned, does its
former owner acquire a cause of action for recovery of the property? Notwithstanding, the lots were subsequently
expropriated under eminent domain. Subsequently, when the Mactan International Airport commenced operations, the
Lahug Airport was ordered closed by then President Corazon C. Aquino. On August 1992, Apolonio Gopuco, Jr. filed for
recovery of the lot, by virtue of the public purpose for which the land was seized having ceased. MCIAA replied that
there has been no express stipulation on the expropriation proceedings regarding reconveyance.

Issues:
1. When private land is expropriated for a particular public use in fee simple, and that particular public
use is abandoned, does its former owner acquire a cause of action for recovery of the property?

Ruling:
1. NO. If, for example, land is expropriated for a particular purpose, with the condition that when that
purpose is ended or abandoned the property shall return to its former owner, then, of course, when the purpose is
terminated or abandoned the former owner reacquires the property so expropriated. However, if upon the contrary, the
decree of expropriation gives to the entity a fee simple title, then of course, the land becomes the absolute property of
the expropriator, whether it be the State, a province, or municipality, and in that case the non-user does not have the
effect of defeating the title acquired by the expropriation proceedings. In the case at bar, there was no such agreement
on the right to reconveyance.

Important note:
The SC deemed it fair to elucidate that the difference between this and the other cases involving Lahug Airport
(Particularly the Heirs of Moreno case), is that the complainants therein either appealed the expropriation, or signed a
compromise agreement with the State conferring the right to repurchase. Apolonio Gopuco in this case, did neither.
Reyes v. National Housing Authority
[EMINENT DOMAIN]

Facts:
Petitioner Reyes owns sugarland lots in Dasmariñas, Cavite. The land was expropriated by the
Dasmariñas resettlement project as a means of accommodating the scores of squatters displaced from their shelters in
Manila. Petitioners were paid just compensation. The NHA however, failed to comply with this public objective, and
instead entered into a contract with a real estate developer for the construction of low cost housing on the expropriated
lots. Petitioner filed for re-acquisition of the land.

Issues:
1. Does the NHA have a duty to return the assailed land to petitioner?

Ruling:
1. NO. When land has been acquired for public use in fee simple unconditionally, either by the exercise
of eminent domain or by purchase, the former owner retains no rights in the land, and the public use may be
abandoned, or the land may be devoted to a different use, without any impairment of the estate or title acquired, or any
reversion to the former owner. Nevertheless, the low cost housing project of respondent NHA is in fact in compliance,
albeit indirectly, of the public purpose requirement, because the Constitution itself allows the State to undertake, for the
common good and in cooperation with the private sector, a continuing program of urban land reform and housing which
will make at affordable cost decent housing and basic services to underprivileged and homeless citizens in urban
centers and resettlement areas.
Philippine Rural Electric Cooperatives Association, Inc. v. DILG
[EQUAL PROTECTION / NON-IMPAIRMENT CLAUSE]

Facts:
Under P.D. No. 269, as amended, or the National Electrification Administration Decree, it is the declared
policy of the State to provide the total electrification of the Philippines on an area coverage basis the same being vital to
the people and the sound development of the nation. Pursuant to the goals set forth by this law, the government
entered into loan agreements with the U.S. government to fund Philippine electric cooperatives. As such, petitioners
believe that they enjoy tax exemptions by virtue of the wording of the agreement. With the passage of the Local
Government Code however, PHILRECA claims that their exemptions have been invariably withdrawn.

Issues:
1. Does the Local Government Code violate the PHILRECA’s right against the impairment of obligations
of contracts?
2. Does the Code violate the equal protection clause?

Ruling:
1. NO. A simple reading of Section of the provisions on taxes and duties in all six loan agreements
readily show that it does not grant tax exemption in favor of the borrower or the beneficiary of the proceeds of the loan.
Instead, it merely shift the tax burden on the transactions under the loan agreements to the borrower and/or the
beneficiary of the loan. Therefore, there is no impairment of the obligation of the borrower, lender, or the beneficiary.
2. NO. Petitioners argue that the above provisions of the Local Government Code are unconstitutional for
violating the equal protection clause. Allegedly, said provisions unduly discriminate against petitioners who are duly
registered cooperatives under P.D. No. 269, as amended, and not under R.A. No. 6938 or the Cooperative Code of the
Philippines. However, the Supreme Court ruled that indeed, there is a reasonable classification that justifies the different
tax treatment between the electric cooperatives covered by the two laws. For one, in the case of cooperatives under
P.D. 269, it is the government that puts in the capital, and the cooperatives themselves are not required to make
equitable contributions. However, under the Cooperative Code, the articles of cooperation of a cooperative applying for
registration must be accompanied with the bonds of the accountable officers and a sworn statement of the treasurer
elected by the subscribers showing that at least twenty-five per cent (25%) of the authorized share capital has been
subscribed and at least twenty-five per cent (25%) of the total subscription has been paid and in no case shall the paid-
up share capital be less than Two thousand pesos (P2,000.00).
Re: Conviction of Judge Adoracion G. Angeles
[RIGHTS OF THE ACCUSED – PRESUMPTION OF INNOCENCE]

Facts:
Respondent judge was convicted in the trial court for violation of R.A. 7610, for alleged counts of child
abuse. Senior State Prosecutor Emmanuel Velasco asked then Chief Justice Panganiban if it was possible to order the
immediate suspension of the respondent. The Court recommended her indefinite suspension, in effect suspending her
from performing her judicial functions while awaiting the final decision of her criminal cases under appeal. Thus,
respondent submitted that her suspension is essentially unjust. Moreover, respondent manifested that the two criminal
cases against her are on appeal before the CA and have, therefore, not yet attained finality. As such, respondent still
enjoys the constitutional presumption of innocence and her suspension clashes with this presumption and is tantamount
to a prejudgment of her guilt.

Issues:
1. Is the aforementioned suspension in violation of the right of the accused to be presumed innocent until
proven guilty?

Ruling:
1. YES. The fact that respondent’s criminal case is still pending appeal should be a bar to her
suspension, because her conviction has not yet attained finality. Until the accused’s guilt is shown beyond reasonable
doubt, the presumption of innocence enjoyed by all those who might be accused of a crime cannot be waived.
Moreover, it is established that any administrative complaint leveled against a judge must always be examined with a
discriminating eye, for its consequential effects are, by their nature, highly penal, such that the respondent judge stands
to face the sanction of dismissal or disbarment.
People v. Rapeza y Francisco
[RIGHTS OF THE ACCUSED – THE MIRANDA DOCTRINE]

Facts:
In two separate informations, Rapeza, along with Mike Regino, were charged with the murder of Spouses
Cesar Ganzon and Priscilla Libas. Upon information supplied by a certain Mr. Dela Cruz that appellant had wanted to
confess to the crimes, SPO2 Gapas invited appellant in for questioning. So, after being detained for one night, accused
was delivered at the house of one Atty. Roberto Reyes, and in the presence of the Vice Mayor of Culion, two officials of
the Sangguniang Barangay, and an interpreter, Gapas proceeded with the custodial investigation, without explicitly
telling the accused of his rights, whereby he extracted the extrajudicial confession from the accused. By virtue of this
confession, the RTC convicted him. Rapeza appeals, claiming that his extrajudicial confession cannot be deemed
admissible.

Issues:
1. Is the extrajudicial confession obtained admissible in court?

Ruling:
1. NO. It is important to note that the appellant was “invited” by SPO2 Gapas to the police station, where
he was detained from 11:00 a.m. until morning the next day, and only then was his extrajudicial confession was
allegedly taken. At this point, the accused should have already been appraised of his constitutional rights, as he was
already considered a suspect. Custodial investigation refers to the critical pre-trial stage when the investigation ceases
to be a general inquiry into an unsolved crime but has begun to focus on a particular person as a suspect. Therefore,
the rights of the accused in a custodial investigation has already begun even before questions were propounded in the
house of Atty. Reyes.
People v. Ting Lan Uy, Jr.
[RIGHTS OF THE ACCUSED – DUE PROCESS IN CRIMINAL PROCEEDINGS / THE MIRANDA DOCTRINE]

Facts:
For allegedly diverting and collecting funds of the National Power Corporation (NPC) intended for the
purchase of US Dollars from the United Coconut Planters Bank (UCPB), Jose Ting Lan Uy, Jr., Ernesto Gamus, Jaime
Ochoa and Raul Gutierrez were indicted before the Sandiganbayan for the complex crime of Malversation through
Falsification of Commercial Document under the Revised Penal Code. The Sandiganbayan found him guilty of
Malversation through inexcusable negligence. Appellant insists that he could not be convicted under the allegations in
the information without violating his constitutional right to due process and to be informed of the accusation against him.
He points out that the information alleges willful and intentional commission of the acts complained of while the
judgment found him guilty of inexcusable negligence amounting to malice.

Issues:
1. Is there a violation of the rights of the accused in this case?
2. Appellant also claims hat he should be acquitted since his conviction was based on his sworn
statement, transcript of stenographic notes from which the sworn statement was taken and the NBI Report, which are
incompetent evidence. He contends that his sworn statement was taken without the benefit of counsel, in violation of his
constitutional right under Section 12, Article III of the 1987 Constitution. Is this contention correct?

Ruling:
1. NO. Malversation may be committed either through a positive act of misappropriation (dolo), or
passively through negligence (culpa). To sustain a charge of malversation, there must either be criminal intent or
criminal negligence. Therefore, even if the prevailing facts of the case may not show that deceit attended the
commission of an offense, it will not prevent the court from accepting evidence to prove that negligence attended the
crime, because both are equally punishable under Article 217 of the Revised Penal Code.
2. NO. The investigation under the above-quoted provision refers to a custodial investigation where a
suspect has already been taken into police custody and the investigating officers begin to ask questions to elicit
information and confessions or admissions from the suspect. Such rights enumerated by the constitutional provision
invoked by accused-appellant are not available before government investigators enter the picture. Thus, the flaw in
appellants argument in this regard becomes immediately apparent vis--vis the foregoing legal yardsticks, considering
that his statement was taken during the administrative investigation of NPCs audit team and before he was taken into
custody. As such, the inquest was still a general inquiry into an unsolved offense at the time and there was, as yet, no
specific suspect.
People v. Baloloy
[RIGHTS OF THE ACCUSED – THE MIRANDA DOCTRINE]

Facts:
Juanito Baloloy was charged with the crime of rape with homicide, and was subsequently convicted. In
his appeal, he claims that the Court erred in admitting in evidence his alleged extrajudicial confession made before
Barangay Captain Ceniza and Judge Dicon. According to him, the two failed to inform him of his constitutional rights
before they took it upon themselves to elicit from him the incriminatory information. It is of no moment that Ceniza and
Dicon are not police investigators, for as public officials it was incumbent upon them to observe the express mandate of
the Constitution. While these rights may be waived, the prosecution failed to show that he effectively waived his rights
through a written waiver executed in the presence of counsel. He concludes that his extrajudicial confession is
inadmissible in evidence.

Issues:
1. Is there a violation of the rights of the accused in this case?

Ruling:
1. NO. It has been held that the constitutional provision on custodial investigation does not apply to a
spontaneous statement, not elicited through questioning by the authorities but given in an ordinary manner whereby the
suspect orally admits having committed the crime. Neither can it apply to admissions or confessions made by a suspect
in the commission of a crime before he is placed under investigation. What the Constitution bars is the compulsory
disclosure of incriminating facts or confessions. The rights under Section 12 of the Constitution are guaranteed to
preclude the slightest use of coercion by the state as would lead the accused to admit something false, not to prevent
him from freely and voluntarily telling the truth. In the instant case, after he admitted ownership of the black rope and
was asked by Ceniza to tell her everything, Juanito voluntarily narrated to Ceniza that he raped Genelyn and thereafter
threw her body into the ravine. This narration was a spontaneous answer, freely and voluntarily given in an ordinary
manner. It was given before he was arrested or placed under custody for investigation in connection with the
commission of the offense.
People v. Zuela
[RIGHTS OF THE ACCUSED – THE MIRANDA DOCTRINE]

Facts:
Zuela, along with two others, were charged with robbery with homicide. On June 4, and 9, 1985, Tito
and Nelson were taken into police custody without a warrant. They underwent custodial investigation without the
assistance of counsel because no lawyer could be found in Cabusao, Camarines Sur. On the last page of each
accused's confession appeared a statement, in their own handwriting, to the effect that they voluntarily gave their
statements and that no one coerced or promised them anything to admit responsibility for the crime. Accused however,
interposed the defense that they were tortured and forced to make a confession.

Issues:
1. Is the extrajudicial confession made by the accused admissible in court?

Ruling:
1. NO. The evidence cannot be admissible, for they were executed without the assistance of counsel.
Despite the fact that the reason for the absence of lawyer during the custodial investigation was the scarcity of lawyers
in the area, the Court could not be lenient in this case. The absence or scarcity of lawyers in any given place is not a
valid reason for defying the constitutional mandate on counseled confessions. Contrary to the ruling of the trial court,
the defect in the confessions of Tito and Nelson was not cured by their signing the extra-judicial statements before
Judge Bagalacsa.

Important note:
Nevertheless, the infirmity of accused-appellants' sworn statements did not leave a void in the prosecution's case.
Accused-appellant Maximo repeated the contents of his sworn statement to Romualda Algarin who, in turn, related
these in court. Such declaration to a private person is admissible in evidence against accused-appellant Maximo
pursuant to Rule 130, Section 26 of the Rules of Court stating that the "act, declaration for omission of a party as to a
relevant fact may be given in evidence against him."
People v. Bermas
[RIGHTS OF THE ACCUSED – THE MIRANDA DOCTRINE]

Facts:
Accused Rufino Mirandilla Bermas has been charged with raping his own daughter. Throughout the
pendency of the criminal proceedings, the accused was represented by Atty. Villarin, who would soon inexplicably waive
the cross-examination on the first prosecution witness and asked the court to be relieved of her duty as counsel de
officio. To substitute for her, the Public Attorney recommended Atty. Roberto Gomez to be appointed as defense
counsel de officio. Upon asking for a continuance however, the latter never surfaced in the courtroom again. Because of
this, Atty. Nicanor Lonzame was appointed as counsel de officio. When the rescheduled date arrived, Atty. Lonzame
aked to be relieved as counsel de officio, then immediately withdrew his statement. Nevertheless, he also ceased to
appear for and in behalf of the accused.

Issues:
1. Is there a violation of the rights of the accused in this case?

Ruling:
1. YES. The Supreme Court stated that a defendant in a criminal proceeding is entitled to be represented
by counsel at every stage of the criminal proceeding. So important is the right to counsel that it has been enshrined in
our fundamental law and its precursor laws. The constitutional mandate is reflected in the 1985 Rules of Criminal
Procedures which declares in Section 1, Rule 115, thereof, that it is a right of the accused at the trial to be present in
person and by counsel at every stage of the proceedings from the arraignment to the promulgation of the judgment. The
court remanded this case to the court a quo for trial, appointing Atty. Ricardo A. Fernandez, Jr. of the Anti-Death
Penalty Task Force as the counsel de officio of the accused.
People v. Tulin
[RIGHTS OF THE ACCUSED – THE MIRANDA DOCTRINE]

Facts:
In May of 1991, Roger Tulin, along with other accused, were arrested and charged with acts of qualified
piracy. Accused-appellants Tulin, Loyola, Infante, Jr., and Cecilio Changco assert that the trial court erred in allowing
them to adopt the proceedings taken during the time they were being represented by Mr. Tomas Posadas, a non-
lawyer, thereby depriving them of their constitutional right to procedural due process. In this regard, said accused-
appellants narrate that Mr. Posadas entered his appearance as counsel for all of them. However, in the course of the
proceedings, or on February 11, 1992, the trial court discovered that Mr. Posadas was not a member of the Philippine
Bar. This was after Mr. Posadas had presented and examined seven witnesses for the accused.

Issues:
1. Is there violation of rights of the accused under custodial investigation in this case?

Ruling:
1. NO. The facts of the case reveals that a manifestation was executed by accused-appellants, stating
that they were adopting evidence adduced when they were represented by a non-lawyer. Moreover, when they were
represented by Atty. Abdul Basar in trial, they made a categorical manifestation that they were apprised of the nature
and legal consequences of subject manifestation, and that they voluntarily and intelligently executed the same. They
also affirmed the truthfulness of its contents when asked in open court . These overt acts constitute a valid waiver of the
right to sufficient representation during the trial, considering that it was unequivocally, knowingly, and intelligently made
and with the full assistance of a bona fide lawyer.
People v. Larrañaga
[RIGHTS OF THE ACCUSED – THE DEATH PENALTY]

Facts:
Larrañaga, Aznar, Adlawan, Caño and Balansag among other accused, were charged with the case of
serious illegal detention with homicide and rape. Among the issues heretofore is the constitutionality of R.A. 7659 which
imposes the death penalty for heinous crimes. In this case, three justices of the Supreme Court maintain their position
that RA 7659 is unconstitutional insofar as it prescribes the death penalty; nevertheless, they submit to the ruling of the
majority that the law is constitutional and the death penalty can be lawfully imposed in the case at bar.

Issues:
1. Should the death penalty be imposed in this case?

Ruling:
1. NO. The 1987 Constitution provides for the prohibition against the imposition from death penalty
through Section 19, Article III, which states: “…Neither shall death penalty be imposed, unless, for compelling reasons
involving heinous crimes, the Congress hereafter provides for it. Any death penalty already imposed shall be reduced
to reclusion perpetua.” However, in the case at bar, this question was rendered moot and academic, because the
Supreme Court appreciated the defense of minority, as the Uy brothers claimed that James Andrew was only seventeen
(17) years and two hundred sixty two (262) days old at the time the crimes were committed. In light of this defense,
Article 68 of the Revised Penal Code, which mandates that the penalty to be imposed upon a person over fifteen and
under eighteen years of age is the penalty next lower than that prescribed by law shall be imposed. By this alone, the
penalty of death has already been reduced to reclusion perpetua.
Teves v. Sandiganbayan
[RIGHTS OF THE ACCUSED – DUE PROCESS IN CRIMINAL PROCEEDINGS]

Facts:
Edgar Teves, former Mayor of Valencia, Negros Oriental, and his wife, was convicted by the
Sandiganbayan for violation of Section 3(h) of Republic Act No. 3019, otherwise known as the Anti-Graft and Corrupt
Practices Act, for unlawful intervention, in his official capacity, on the issuance of a license in favor of a business
enterprise in which he had a pecuniary interest. Petitioners allege that their right to be informed of the nature and cause
of the accusation against them was violated, because while the charge was for alleged intervention in the issuance of a
cockpit license, they were instead convicted of having a direct financial or pecuniary interest prohibited under Section
89(2) of the Local Government Code.

Issues:
1. In the case at bar, did the difference in the offense charged and that proved constitute a violation of
criminal due process?

Ruling:
1. NO. A reading of both Section 3(h) of the Anti-Graft Law and Section 89(2) of the LGC will reveal that
in both offenses, two of the three essential elements constituting the offense are exactly the same, and in fact, it is
these two elements that were proven during the trial. Hence, the offense proved is necessarily included in the offense
charged, or the offense charged necessarily includes the offense proved. The variance doctrine thus finds application to
this case, thereby warranting the conviction of petitioner Edgar Teves for the offense proved.
Estrada v. People
[RIGHTS OF THE ACCUSED – DUE PROCESS IN CRIMINAL PROCEEDINGS]

Facts:
An Information charging petitioner with estafa was filed with the RTC of Las Piñas City. In view of the
fact that petitioner jumped bail, the RTC issued an Order dated May 14, 1997, considering petitioner to have waived her
right to present evidence. Thus, the RTC rendered judgment based only on prosecution evidence. On December 1,
1999, petitioner moved for reconsideration and/or new trial stating that her constitutional rights to be heard and to
counsel were violated, because counsel for petitioner was not served a copy of the Order dated March 31, 1997 citing
her for contempt of court, nor was she served any copy of the Order dated May 14, 1997 declaring petitioner to have
waived her right to present evidence.

Issues:
1. Was there a violation of the rights of the accused to due process in this case?

Ruling:
1. NO. At the outset, the undisputed fact that petitioner jumped bail while trial was pending should be
emphasized. By the mere fact that she jumped bail and could no longer be found, petitioner is considered to have
waived her right to be present at the trial, and she and her counsel were to be deemed to have received notice. Due
process is satisfied when the parties are afforded a fair and reasonable opportunity to explain their respective sides of
the controversy.
Martinez v. Mendoza
[RIGHTS OF THE ACCUSED – HABEAS CORPUS]

Facts:
Petitioners are the mother and wife, respectively, of Michael Martinez, a resident of 4570 Cattleya Road,
Sun Valley Subdivision, Paraaque City, who was allegedly abducted and taken away by seven (7) persons around 7:30
in the morning of November 19, 2001 while he was walking along Magnolia Street. The abduction was reported by
petitioners to the Barangay, the Paraaque Police and the Anti-Kidnapping Task Force at Camp Crame. A witness for the
Nida Blanca case, in a television interview, narrated that he saw Michael Martinez detained by the Criminal Investigation
and Detection Group (CIDG) in Camp Crame. Because of this, petitioners asked the DILG to see Martinez. After their
petition was denied, they filed a formal petition for habeas corpus with the Quezon City RTC, which ruled in behalf of
herein petitioners. However, the CA ruled in favor of the OSG, banking on the presumption of regularity, and after
finding the credibility of Medel, who kept contradicting himself during questioning, questionable.

Issues:
1. Should the petition for habeas corpus prevail?

Ruling:
1. NO. In the immediate case, by virtue of the presumption of regularity, the Court has to appreciate that
the DILG’s fervent denial that they have Martinez in their custody is not laden with ill motives. At the outset, it must be
stressed that petitioners anchor for the present case is the disappearance of Michael. The matter of his alleged
detention is, at best, merely consequential to his disappearance. Ostensibly, his disappearance has been established.
However, the grant of relief in a habeas corpus proceeding is not predicated on the disappearance of a person, but on
his illegal detention. Habeas corpus generally applies to all cases of illegal confinement or detention by which any
person is deprived of his liberty or by which the rightful custody of any person is withheld from the person entitled
thereto.
Ilagan v. Enrile
[RIGHTS OF THE ACCUSED – HABEAS CORPUS]

Facts:
This is a petition for Habeas Corpus filed by the Integrated Bar of the Philippines (IBP), the Free Legal
Assistance Group (FLAG); and the Movement of Attorneys for Brotherhood, Integrity and Nationalism (MABINI) on
behalf of Attorneys Laurente C. Ilagan, Antonio B. Arellano, and Marcos Risonar, Jr., who were arrested in Davao City
by elements of the PC-INP and detained on the basis of a mission order allegedly issued by the Ministry of National
Defense. Respondents claim that due to the state of rebellion in Davao City at the time, as well as the presence of
subversive documents confiscated in the persons of the aforementioned attorneys, implied that they played an active
role in organizing mass actions of the Communist Party of the Philippines, therefore the writ of habeas corpus was
suspended to them. Nevertheless, in the midst of proceedings, respondents filed a motion wherein they manifested that
an information for rebellion has already been filed against the detained attorneys before the RTC of Davao.

Issues:
1. Should the petition for habeas corpus prevail over the filing of an information for rebellion?

Ruling:
1. NO. As contended by respondents, the petition herein has been rendered moot and academic by
virtue of the filing of an Information against them for Rebellion, a capital offense, before the Regional Trial Court of
Davao City and the issuance of a Warrant of Arrest against them. The function of the special proceeding of habeas
corpus is to inquire into the legality of one's detention. Now that the detained attorneys' incarceration is by virtue of a
judicial order in relation to criminal cases subsequently filed against them before the Regional Trial Couravao City, the
remedy of habeas corpus no longer lies. The Writ had served its purpose.
Razon v. Tagitis
[RIGHTS OF THE ACCUSED – WRIT OF AMPARO]

Facts:
Tagitis, a consultant for the World Bank and the Senior Honorary Counselor for the Islamic Development
Bank (IDB) Scholarship Programme, was last seen in Jolo, Sulu. Together with Arsimin Kunnong (Kunnong), an IDB
scholar, Tagitis arrived in Jolo by boat in the early morning of October 31, 2007 from a seminar in Zamboanga City.
They immediately checked-in at ASY Pension House. Tagitis asked Kunnong to buy him a boat ticket for his return trip
the following day to Zamboanga. When Kunnong returned from this errand, Tagitis was no longer around. More than a
month later (on December 28, 2007), the respondent (the missing subject’s wife) filed a Petition for the Writ of Amparo
(petition) with the CA. The petition was directed against various officers of the ARMM-PNP, CIDG, and the Anti-Crime
and Emergency Response team of the PNP. The affidavit of PNP Chief Gen. Avelino Razon stated that he did not have
any personal knowledge of, or any participation in, the alleged disappearance; that he had been designated by
President Gloria Macapagal Arroyo as the head of a special body called TASK FORCE USIG, to address concerns
about extralegal killings and enforced disappearances. All respondents likewise denied having knowledge of the
disappearance.

Issues:
1. Is the issuance of a Writ of Amparo proper in this case?

Ruling:
1. YES. The Supreme Court noted that the fact that respondents, instead of initiating a thorough, in-
depth investigation that credibly determined the fate of Tagitis, were instead consumed in consistent and unfounded
denials of their involvement, was an unequivocal proof of some government complicity in the disappearance. For why
would the government and its officials engage in their chorus of concealment if the intent had not been to deny what
they already knew of the disappearance?
The Writ of Amparo is a protective remedy against violations or threats of violation against the rights to
life, liberty, and security of persons. In its present form, the writ is a remedy against enforced disappearances. It does
not determine guilt nor pinpoint criminal culpability for the disappearance; rather, it determines responsibility, or at least
accountability, for the enforced disappearance for purposes of imposing the appropriate remedies to address the
disappearance.

Important note:
Read the full case for a comprehensive discussion on the Writ of Amparo.
Reyes v. Court of Appeals
[RIGHTS OF THE ACCUSED – WRIT OF AMPARO]

Facts:
Petitioner was among those arrested in the Manila Peninsula Hotel siege on November 30, 2007. In the
morning of November 30, 2007, petitioner together with fifty (50) others, were brought to Camp Crame to await inquest
proceedings. On December 1, 2007, upon the request of the Department of Interior and Local Government (DILG),
respondent DOJ Secretary Raul Gonzales issued Hold Departure Order (HDO) No. 45 ordering respondent
Commissioner of Immigration to include in the Hold Departure List of the Bureau of Immigration and Deportation (BID)
the name of petitioner and 49 others relative to the aforementioned case in the interest of national security and public
safety. On December 13, 2007, the RTC issued an Order dismissing the charge for Rebellion against petitioner and 17
others for lack of probable cause, as they were not found to be in conspiracy with the members of the coup. However,
when petitioner attempted to travel abroad, he found out that the HDO was still in effect. He petitioned the court on the
ground of the Writ of Amparo, whereby his right to travel was violated.

Issues:
1. Is the issuance of a Writ of Amparo proper in this case?

Ruling:
1. NO. The rights that fall within the protective mantle of the Writ of Amparo under Section 1 of the Rules
thereon are the following: (1) right to life; (2) right to liberty; and (3) right to security. Section 1 of the Rule on the Writ
states that the petition for a Writ of Amparo is a remedy available to any person whose right to life, liberty and security is
violated or threatened with violation by an unlawful act or omission of a public official or employee, or of a private
individual or entity. Here, the restriction on petitioner’s right to travel as a consequence of the pendency of the criminal
case filed against him was not unlawful. Petitioner has also failed to establish that his right to travel was impaired in the
manner and to the extent that it amounted to a serious violation of his right to life, liberty and security, for which there
exists no readily available legal recourse or remedy.
Secretary of National Defense & AFP Chief of Staff v. Manalo
[RIGHTS OF THE ACCUSED – WRIT OF AMPARO]

Facts:
Brothers Raymond and Reynaldo Manalo, farmers based in Bulacan, were forcibly taken from their
homes by members of the military, led by General Palparan, for being alleged members of the NPA. After 18 months of
detention, they managed to escape, thereafter they filed a petition for Prohibition, Injunction, and Temporary Restraining
Order to prevent the military from depriving them of their basic human and constitutional rights. On December 26, 2007,
the Court of Appeals granted the privilege of the Writ of Amparo.

Issues:
1. Is the issuance of a Writ of Amparo proper in this case?

Ruling:
1. YES. The right to the security of person is not an empty statement in the 1987 Constitution. It is one of
the most fundamentally-protected rights of the person, whether citizen or alien. At its core is the immunity of one’s
person against government intrusion. The right to security of person is “freedom from fear,” a guarantee of bodily and
psychological integrity and security. The law thus gives the remedy of the Writ of Amparo, in addition to the rights and
liberties already protected by the Bill of Rights. Amparo, literally meaning “to protect,” is borne out of the long history of
Latin American and Philippine human rights abuses—often perpetrated by the armed forces against farmers thought to
be communist insurgents, anarchists or brigands. The writ serves to both prevent and cure extralegal killings, enforced
disappearances, and threats thereof, giving the powerless a powerful remedy to ensure their rights, liberties, and
dignity.

Important note:
This is the very first case where the Writ of Amparo was accepted in Philippine jurisprudence.
Bank of the Philippine Islands v. Casa Montessori Internationale
[RIGHTS OF THE ACCUSED – DUE PROCESS IN CRIMINAL PROCEEDINGS]

Facts:
Casa Montessori International opened an account with BPI, with CASA’s President as one of its
authorized signatories. It discovered that 9 of its checks had been encashed by a certain Sonny D. Santos whose name
turned out to be fictitious, and was used by a certain Yabut, CASA’s external auditor. In the investigation conducted by
herein respondent, he voluntarily admitted that he forged the signature and encashed the checks.

Issues:
1. Was the voluntary admission in this case admissible in court proceedings?

Ruling:
1. YES. In the first place, he was not under custodial investigation. is Affidavit was executed in private
and before private individuals. The mantle of protection under Section 12 of Article III of the 1987 Constitution covers
only the period from the time a person is taken into custody for investigation of his possible participation in the
commission of a crime or from the time he is singled out as a suspect in the commission of a crime although not yet in
custody. Therefore, to fall within the ambit of Section 12, quoted above, there must be an arrest or a deprivation of
freedom, with questions propounded on him by the police authorities for the purpose of eliciting admissions,
confessions, or any information. The said constitutional provision does not apply to spontaneous statements made in a
voluntary manner whereby an individual orally admits to authorship of a crime.
People v. Olvis
[RIGHTS OF THE ACCUSED – DUE PROCESS IN CRIMINAL PROCEEDINGS / PROTECTION AGAINST SELF-
INCRIMINATION]

Facts:
Olvis, along with three other accused, was charged with the murder of one Discredit Bagon. While in
custody, the three executed five separate written confessions each. Eventually, Olvis was acquitted, and the remaining
accused-appellants subsequently repudiated their alleged confessions in open court alleging threats by the Polanco
investigators of physical harm if they refused to "cooperate" in the solution of the case. They likewise alleged that they
were instructed by the Polanco police investigators to implicate Anacieto Olvis in the case.

Issues:
1. Was the assailed coerced confessions in this case admissible in court proceedings?

Ruling:
1. NO. The confessions in the case at bar suffer from a Constitutional infirmity. In their supposed
statements dated September 9, 14, and 21, 1975, the accused-appellants were not assisted by counsel when they
"waived" their rights to counsel. But the accused-appellants were denied their right to counsel not once, but twice, as
they were forced to re-enact the crime shortly after they were apprehended. Forced re-enactments, like uncounselled
and coerced confessions come within the ban against self- incrimination. This constitutional privilege has been defined
as a protection against testimonial compulsion, but this has since been extended to any evidence "communicative in
nature" acquired under circumstances of duress. Essentially, the right is meant to "avoid and prohibit positively the
repetition and recurrence of the certainly inhuman procedure of competing a person, in a criminal or any other case, to
furnish the missing evidence necessary for his conviction."
Disini v. Sandiganbayan
[NON-IMPAIRMENT CLAUSE]

Facts:
In 1989 respondent Republic of the Philippines, represented in this case by the Presidential Commission
on Good Government (PCGG), wanted petitioner Jesus P. Disini to testify for his government in its case against
Westinghouse Electric Corporation (specifically against his second cousin, Herminio Disini, whom he worked for) before
the United States District Court of New Jersey. On February 16, 1989 respondent Republic and petitioner Disini entered
into an Immunity Agreement under which Disini undertook to testify for his government and provide its lawyers with the
information, affidavits, and documents they needed for prosecuting the two cases, and in turn the government shall not
compel Disini to testify in any other domestic or foreign proceeding brought by the Republic against Herminio. 18 years
later, respondent issued a subpoena duces tecum and ad testificandum against Disini, commanding him to testify and
produce documents before that court on March 6 and 30, 2007 in an action that the Republic filed against Herminio.

Issues:
1. May the Sandiganbayan compel petitioner to testify, thus impairing the immunity agreement he signed
18 years prior?

Ruling:
1. NO. A contract is the law between the parties. It cannot be withdrawn except by their mutual consent.
This applies with more reason in this case where petitioner Disini had already complied with the terms and conditions of
the Immunity Agreement. To allow the Republic to revoke the Agreement at this late stage will run afoul of the rule that
a party to a compromise cannot ask for a rescission after it had enjoyed its benefits. Moreover, the grant of immunity in
the Agreement to petitioner Disini against being compelled to testify in "other cases" against Herminio is quite clear and
does not need any interpretation. Where a stipulation in an agreement is clear, its literal meaning controls.
Social Justice Society v. Dangerous Drugs Board
[RIGHTS OF THE ACCUSED – PROTECTION AGAINST SELF-INCRIMINATION]

Facts:
This petition assails the constitutionality of Section 36 of the Comprehensive Dangerous Drugs Act,
insofar as it requires mandatory drug testing of candidates for public office, students of secondary and tertiary schools,
officers and employees of public and private offices, and persons charged before the prosecutors office with certain
offenses, among other personalities.

Issues:
1. Is a mandatory drug test in violation of the rights of the accused against self-incrimination?

Ruling:
1. YES. The Supreme Court finds no valid justification for mandatory drug testing for persons accused of
crimes. In the case of students, the constitutional viability of the mandatory, random, and suspicionless drug testing for
students emanates primarily from the waiver by the students of their right to privacy when they seek entry to the school,
and from their voluntarily submitting their persons to the parental authority of school authorities. In the case of private
and public employees, the constitutional soundness of the mandatory, random, and suspicionless drug testing proceeds
from the reasonableness of the drug test policy and requirement. However, in the case of persons charged with a crime
before the prosecutors office, a mandatory drug testing can never be random or suspicionless. The ideas of
randomness and being suspicionless are antithetical to their being made defendants in a criminal complaint. They are
not randomly picked; neither are they beyond suspicion. When persons suspected of committing a crime are charged,
they are singled out and are impleaded against their will. The persons thus charged, by the bare fact of being haled
before the prosecutors office and peaceably submitting themselves to drug testing, if that be the case, do not
necessarily consent to the procedure, let alone waive their right to privacy.
Herrera v. Sandiganbayan
[RIGHTS OF THE ACCUSED – DUE PROCESS IN CRIMINAL PROCEEDINGS]

Facts:
Petitioners Pat. Edgardo Herrera y Baltoribio and Pat. Redentor Mariano y Antonio, together with the
other accused, Pat. Roberto Barrera and Pat. Rodolfo Alcalde, all members of the Parañaque Police Station, were
charged with two (2) counts of murder, for the killing of Shi Shu Yang and George Go y Tan before the Sandiganbayan.
On December 13, 1994, respondent Sandiganbayan convicted each of the petitioners of two counts of murder.
Petitioners contend that, among other reasons, the respondent tribunal should have allowed their counsel to conduct
further cross-examination on the prosecution witnesses.

Issues:
1. Is the aforementioned act of respondent in violation of the right of the accused to meet the witnesses
face-to-face?

Ruling:
1. NO. According to Section 6 of Rule 132 of the Revised Rules on Evidence, the witnesses may, upon
termination of the direct examination, be cross-examined by the adverse party as to any matter stated in the direct
examination, or connected therewith, with sufficient fullness and freedom to test his accuracy and truthfulness and
freedom from interest or bias. Furthermore, Article III, Section 14(2) provides for the right of the accused to meet the
witnesses face-to-face. Rule 115, Section 1(f) of the Revised Rules of Criminal Procedure also states that, in all criminal
prosecutions, the accused shall have the right to confront and cross-examine the witnesses against him. Indeed,
petitioners counsel has conducted an extensive cross-examination of witness Winterhalter on the scheduled dates of
hearing. Petitioners, therefore, cannot claim there has been any procedural infirmity in the proceedings. Moreover, the
trial court has the power to direct the course of the trial either to shorten or to extend the direct or cross examination of
a counsel. Under Rule 133, Section 6 of the Revised Rules on Evidence, the court may stop the introduction of further
testimony upon any particular point when the evidence upon it is already so full that more witnesses to the same point
cannot be reasonably expected to be additionally persuasive.
People v. Dumlao
[RIGHTS OF THE ACCUSED – DOUBLE JEOPARDY]

Facts:
On 19 July 1991, an Amended Information was filed before the Sandiganbayan charging respondents
Dumlao and Lao, Aber P. Canlas, Jacobo C. Clave, Roman A. Cruz, Jr. and Fabian C. Ver with violation of Section 3(g)
of Republic Act No. 3019, as amended, otherwise known as the Anti-Graft and Corrupt Practices Act. After his
arraignment and plea of not guilty, respondent Dumlao filed a motion to dismiss on the ground that the charges do not
constitute an offense. Accordingly, the Sandiganbayan dismissed the case. The Ombudsman however, filed a petition
for certiorari, seeking the reversal and setting aside of the assailed resolution. Dumlao contends that to give due course
to the petition would constitute double jeopardy.

Issues:
1. Will the Ombudsman violate the constitutional provision against double jeopardy should it approve the
petition?

Ruling:
1. NO. To raise the defense of double jeopardy, three requisites must be present: (1) a first jeopardy
must have attached prior to the second; (2) the first jeopardy must have been validly terminated; and (3) the second
jeopardy must be for the same offense as that in the first. The first jeopardy attaches only (1) upon a valid indictment;
(2) before a competent court; (3) after arraignment; (4) when a valid plea has been entered; and (5) when the defendant
was convicted or acquitted, or the case was dismissed or otherwise terminated without the express consent of the
accused. In the instant case, double jeopardy has not yet set in. The first jeopardy has not yet attached. There is no
question that four of the five elements of legal jeopardy are present. However, there was no valid conviction, acquittal,
dismissal, or termination of the case. The Sandiganbayan dismissed the case for insufficiency of evidence, while the
ground invoked by the respondent was that the facts charged did not constitute an offense. The dismissal was clearly
premature, because any dismissal based on insufficiency of evidence may only be made after the prosecution rests its
case and not at any time before then. A purely capricious dismissal of an information deprives the State of a fair
opportunity to prosecute and convict. It denies the prosecution a day in court. It is void and cannot be the basis of
double jeopardy.
People v. Mojello
[RIGHTS OF THE ACCUSED – THE MIRANDA DOCTRINE]

Facts:
On January 21, 1999, the trial court of Bogo, Cebu, rendered judgment finding appellant guilty beyond
reasonable doubt of the crime of rape with homicide, and sentencing him to suffer the death penalty. Appellant was
arrested at Bantayan while attempting to board a motor launch bound for Cadiz City. On an investigation conducted by
SPO2 Wilfredo Giducos, he admitted that he was the perpetrator of the dastardly deed. Appellant was assisted by Atty.
Isaias Giduquio during his custodial interrogation. His confession was witnessed by Barangay Captains Wilfredo
Batobalanos and Manolo Landao. Batobalanos testified that after it was executed, the contents of the document were
read to appellant who later on voluntarily signed it. Appellant's extrajudicial confession was sworn before Judge Cornelio
T. Jaca of the Municipal Circuit Trial Court (MCTC) of Sta. Fe-Bantayan. Petitioner, in his appeal, averred that he was
not knowingly and intelligently appraised of his constitutional rights before his confession was taken from him.

Issues:
1. Is the evidence of extrajudicial confession admissible?

Ruling:
1. NO. At the core of the instant case is the application of the law on custodial investigation enshrined in
Article III, Section 12, paragraph 1 of the Constitution. The extrajudicial confession executed by appellant on December
23, 1996, applying Art. III, Sec. 12, par. 1 of the Constitution in relation to Rep. Act No. 7438, Sec. 2 complies with the
strict constitutional requirements on the right to counsel. In other words, the extrajudicial confession of the appellant is
valid and therefore admissible in evidence.
People v. Taliman
[RIGHTS OF THE ACCUSED – THE MIRANDA DOCTRINE]

Facts:
Pedro Taliman, Basilio Baybayan and Amado Belano were charged with murder. A custodial
investigation was conducted on July 23, 1990, whereby the accused made extrajudicial statements, effectively
confessing to the crime, in the presence of Atty. Nicolas V. Pardo, then Mayor of Labo, Camariñes Norte. It was during
this custodial investigation that accused Basilio Baybayin confessed to prosecution witness Sgt. Bonifacio Argarin that
he participated in the killing of Renato because Renato did not give them the money they were demanding. This
confession was given without the assistance of counsel and was not reduced to writing.

Issues:
1. Is the mayor, who happens to be a lawyer, considered an independent counsel for the accused ion the
custodial investigation?
2. Was there a valid waiver of the right to counsel in this case?

Ruling:
1. NO. Mayor Pardo cannot be considered as an independent counsel for accused during their custodial
investigation. The Supreme Court in People v. Culala ruled that an extrajudicial confession made by the accused was
inadmissible when he was assisted by the incumbent municipal attorney. As legal officer of the municipality, he provides
legal assistance and support to the mayor and the municipality in carrying out the delivery of basic services to the
people, including the maintenance of peace and order. It is therefore seriously doubted whether he can effectively
undertake the defense of the accused without running into conflict of interests.
2. NO. The Constitution expressly provides that the waiver must be in writing and in the presence of
counsel.
People v. Tomaquin
[RIGHTS OF THE ACCUSED – THE MIRANDA DOCTRINE]

Facts:
Accused Tomaquin was found guilty of the crime of murder by the RTC. In his appeal, he averred that in
his custodial investigation where he confessed to the crime, he was assisted by Atty. Fortunato Parawan, then a
barangay captain of Brgy. Lorega, Cebu City, and so his right to competent and independent counsel was violated.

Issues:
1. Is the evidence of extrajudicial confession admissible?

Ruling:
1. NO. The words "competent and independent counsel" in the constitutional provision is not an empty
rhetoric. It stresses the need to accord the accused, under the uniquely stressful conditions of a custodial investigation,
an informed judgment on the choices explained to him by a diligent and capable lawyer. Atty. Parawan, as barangay
captain, is called upon to enforce the law and ordinances in his barangay and ensure peace and order at all times. In
fact, as barangay captain, Atty. Parawan is deemed a person in authority under Article 152 of the Revised Penal Code.
As such, it is not legally possible to consider the latter an independent counsel for appellant.
People v. Santocilles, Jr.
[RIGHTS OF THE ACCUSED – THE MIRANDA DOCTRINE]

Facts:
Santocilles has been convicted for rape. Appellant contends that he was represented during trial by a
person named Gualberto C. Ompong, who for all intents and purposes acted as his counsel and even conducted the
direct examination and cross-examinations of the witnesses. On appeal, however, appellant secured the services of a
new lawyer, Atty. Igmedio S. Prado, Jr., who discovered that Gualberto C. Ompong is actually not a member of the bar.
Further verification with the Office of the Bar Confidant confirmed this fact. Appellant therefore argues that his
deprivation of the right to counsel should necessarily result in his acquittal of the crime charged.

Issues:
1. Has there been a violation of the rights of the accused to counsel?

Ruling:
1. YES. The presence and participation of counsel in criminal proceedings should never be taken lightly.
Even the most intelligent or educated man may have no skill in the science of the law, particularly in the rules of
procedure, and, without counsel, he may be convicted not because he is guilty but because he does not know how to
establish his innocence. Such a right proceeds from the fundamental principle of due process which basically means
that a person must be heard before being condemned. Section 1 of Rule 138 of the Rules of Court explicitly states who
are entitled to practice law in the Philippines, and Section 2 thereof clearly provides for the requirements for all
applicants for admission to the bar. Jurisprudence has also held that "the right to practice law is not a natural or
constitutional right but is in the nature of a privilege or franchise. It is limited to persons of good moral character with
special qualifications duly ascertained and certified.

Important note:
The case was remanded to the RTC for new trial.
Spouses Romualdez v. COMELEC
[RIGHTS OF THE ACCUSED – DUE PROCESS IN CRIMINAL PROCEEDINGS]

Facts:
Petitioners contend that the election offenses for which they are charged by private respondent are
entirely different from those which they stand to be accused of before the RTC by the COMELEC. According to
petitioners, private respondent’s complaint charged them for allegedly violating, to wit: 1) Section 261(y)(2) and Section
261(y)(5) of the Omnibus Election Code, and 2) Section 12 of the Voter’s Registration Act; however, the COMELEC En
Banc directed in the assailed Resolutions, that they be charged for violations of Section 10(g) and (j), in relation to
Section 45(j) of the Voter’s Registration Act. Essentially, petitioners are of the view that they were not accorded due
process of law. Specifically, their right to refute or submit documentary evidence against the new charges which
COMELEC ordered to be filed against them.

Issues:
1. Is there a violation of due process in this case?

Ruling:
1. NO. According to the Supreme Court, the allegations in the Complaint-Affidavit which was filed with
the Law Department of the COMELEC, support the charge directed by the COMELEC En Banc to be filed against
petitioners with the RTC. Even a mere perusal of the Complaint-Affidavit would readily show that Section 10 of Republic
Act No. 8189 was specifically mentioned therein. Petitioners cannot be said to have been denied due process on the
claim that the election offenses charged against them by private respondent are entirely different from those for which
they stand to be accused of before the RTC, as charged by the COMELEC. In the first place, there appears to be no
incongruity between the charges as contained in the Complaint-Affidavit and the Informations filed before the RTC.
Rodriguez v. Presiding Judge of the Manila RTC
[RIGHTS OF THE ACCUSED – DUE PROCESS IN CRIMINAL PROCEEDINGS / BAIL]

Facts:
Petitioner was arrested by virtue of a petition for extradition filed by the U.S. government. After thearrest,
petitioner applied for bail which the trial court granted on September 25, 2001. Unsatisfied with the bail, the US
government filed a petition for certiorari with the Supreme Court, who then remanded the case back to the RTC for
resolution. The RTC, without prior notice and hearing, cancelled the cash bond of the petitioners and ordered the
issuance of a warrant of arrest.

Issues:
1. Is there a violation of due process in this case?

Ruling:
1. YES. As already established in the Puruganan case, a prospective extraditee is not entitled to notice
and hearing before the issuance of a warrant of arrest, because notifying him before his arrest only tips him of his
pending arrest. But this is for cases pending the issuance of a warrant of arrest, not in a cancellation of a bail that had
been issued after determination that the extraditee is a no-flight risk. Given these facts, it is likely that the RTC
exercised grave abuse of discretion in cancelling the bail without due notice and hearing. Considering that she has not
been shown to be a flight risk nor a danger to the community, she is entitled to notice and hearing before her bail could
be cancelled.
Romualdez v. Sandiganbayan
[RIGHTS OF THE ACCUSED – DUE PROCESS IN CRIMINAL PROCEEDINGS]

Facts:
The Presidential Commission on Good Government (PCGG) filed an information before the anti-graft
court on July 12, 1989 charging Romualdez with violation of Sec. 5, Republic Act No. 3019. Romualdez claims, among
others, that the information charged against him are unconstitutionally vague, because it does not specify the acts of
intervention that he supposedly performed.

Issues:
1. Is there a violation of due process in this case, particularly in line with the void-for-vagueness doctrine?

Ruling:
1. NO. The rule merely requires the information to describe the offense with sufficient particularity as to
apprise the accused of what they are being charged with and to enable the court to pronounce judgment. The
particularity must be such that persons of ordinary intelligence may immediately know what is meant by the information.
While it is fundamental that every element of the offense must be alleged in the information, matters of evidence -- as
distinguished from the facts essential to the nature of the offense -- need not be averred. Whatever facts and
circumstances must necessarily be alleged are to be determined by reference to the definition and the essential
elements of the specific crimes.
Estrada v. Sandiganbayan
[RIGHTS OF THE ACCUSED – DUE PROCESS IN CRIMINAL PROCEEDINGS]

Facts:
Former President Joseph Estrada, in light of the plunder cases against him, is assailing the
constitutionality of the Plunder Law (RA 7080) on two grounds: the void-for-vagueness doctrine, and the purported fact
that it requires less evidence for providing the predicate crimes of plunder, in effect violating the rights of the accused to
due process.

Issues:
1. Should RA 7080 be declared unconstitutional for vagueness?
2. Is RA 7080 in violation of the rights of the accused in a criminal case to due process?

Ruling:
1. NO. As long as the law affords some comprehensible guide or rule that would inform those who are
subject to it what conduct would render them liable to its penalties, its validity will be sustained. The amended
information itself closely tracks the language of law, indicating w/ reasonable certainty the various elements of the
offense w/c the petitioner is alleged to have committed. Petitioner further claims that the law is void for it provides vague
definitions for the words “combination” and “series” under the phrase “a combination or series of overt or criminal acts.”
The Supreme Court declared that a statute is not rendered void merely because general terms are used therein, nor
because it uses terms without definition.
2. NO. The use of the "reasonable doubt" standard is indispensable to command the respect and
confidence of the community in the application of criminal law. It has acquired such an exalted stature in constitutional
law, as it gives life to the very concept of due process. However, in this case, the Plunder Law does not, in contrary to
the contention of petitioner, water down this immaculate standard of proof in criminal procedure. What the prosecution
needs to prove beyond reasonable doubt is only a number of acts sufficient to form a combination or series which would
constitute a pattern and involving an amount of at least P50,000,000.00. There is no need to prove each and every
other act alleged in the Information to have been committed by the accused in furtherance of the overall unlawful
scheme or conspiracy to amass, accumulate or acquire ill-gotten wealth.
David v. Macapagal-Arroyo
[SEARCHES AND SEIZURES]

Facts:
On February 24, 2006, President Arroyo issued PP 1017, declaring a state of emergency. Respondent
reasoned out that the proximate cause behind the executive issuances was the conspiracy among some military
officers, leftist insurgents of the New People’s Army, and some members of the political opposition in a plot to unseat or
assassinate President Arroyo. Pursuant to this, petitioners David and Llamas were arrested without warrants on
February 24, 2006 on their way to EDSA, because the arresting officers, a composite team of PNP and AFP officers,
deemed that they were donning subversive signs and symbols (“oust Gloria” t-shirts). Meanwhile, the offices of the
newspaper Daily Tribune, which was perceived to be anti-Arroyo, was searched without warrant at about 1:00 A.M. on
February 25, 2006. Seized from the premises – in the absence of any official of the Daily Tribune except the security
guard of the building – were several materials for publication.

Issues:
1. Were the warrantless searches and arrests made valid?

Ruling:
1. NO. The arrests in the first case falls neither within the provisions of arrest in flagrante delicto, nor hot
pursuit or stop-and-frisk scenarios. In the immediate case, the Prosecutor ordered petitioners’ release, because the
mere act of wearing a t-shirt cannot be grounds for a warrantless arrest for inciting to sedition. Moreover, the arresting
officers cannot invoke PP 1017, because there is nothing in the proclamation allowing the police, expressly or impliedly,
to conduct illegal arrest, search or violate the citizens’ constitutional rights. The search was likewise illegal, because it
was not made without a valid warrant, nor was it made in the presence of the lawful occupant of the house or two
witnesses of sufficient age and discretion residing within the same locality.
Manapat v. Court of Appeals
[EMINENT DOMAIN]

Facts:
In the 1960s, the Roman Catholic Archbishop of Manila allowed a number of individuals to occupy the
Grace Park property on condition that they would vacate the premises should the former push through with the plan to
construct a school in the area. The government, acting on behalf of the occupants who wanted to purchase the portions
they occupied, negotiated for the acquisition of the properties. However, because of the high asking price of RCAM and
the budgetary constraints of the Government, the latter’s effort to purchase and/or to expropriate the property was
discontinued. When the government finally had the necessary funds, an expropriation was made, in effect condemning
the aforementioned properties and compensated the occupants.

Issues:
1. Were the conditions for taking under eminent domain complied with in this case?

Ruling:
1. YES. The following constitute requisites for the valid exercise of the power of eminent domain: (1) the
property taken must be private property; (2) there must be genuine necessity to take the private property; (3) the taking
must be for public use; (4) there must be payment of just compensation; and (5) the taking must comply with due
process of law. As to the second condition, the genuine necessity of taking private property is a political question that
the judiciary cannot encroach upon. The expropriation was made under Presidential Decree No. 1072 by former
President Marcos, who at the time had full legislative reign. As to the third requisite of public use, the government
justifies the taking of the subject property for the purpose of improving and upgrading the area by constructing roads
and installing facilities thereon. Just compensation on the other hand, is a judicial prerogative, and thus cannot be
questioned. As to the observance of the fifth requisite, the due process clause, in the expropriation proceedings, all the
parties have been given their day in court. The existence of this case is in and of itself proof of that.

Important note:
I suggest reading the full case for comprehensive information on the case and its issues.
Republic v. Judge Gingoyon
[EMINENT DOMAIN]

Facts:
The present controversy has its roots with the promulgation of the Courts decision in Agan v. PIATCO,
where the court nullified the Concession Agreement for the Build-Operate-and-Transfer Arrangement of NAIA 3
between the government and the Philippine International Air Terminals Co., Inc. (PIATCO). The agreement had
authorized PIATCO to build a new international airport terminal (NAIA 3), as well as a franchise to operate and maintain
the said terminal during the concession period of 25 years. When the 2003 decision for annulment was released,
though, the ponente was silent as to the rights of reimbursement of PIATCO, as there has already been substantial
performance of its obligation, seeing as NAIA 3 was already nearing completion then. On 21 December 2004, the
government called for the expropriation of NAIA 3 facilities, contending that a deposit of the assessed value of the
property entitled it to a writ of possession. However, private respondents claim that before an entitlement to such a writ
is issued, payment of just compensation must first be made to the builders of the NAIA 3 facilities.

Issues:
1. Should government pay just compensation before it can be issued entitlement to a writ of possession?

Ruling:
1. YES. Rep. Act No. 8974 requires that immediate payment of the initially determined amount of just
compensation should first be effected before the issuance of a writ of possession. Rep. Act No. 8974 is plainly clear in
imposing the requirement of immediate prepayment, and no amount of statutory deconstruction can evade such
requisite. It enshrines a new approach towards eminent domain that reconciles the inherent unease attending
expropriation proceedings with a position of fundamental equity. While expropriation proceedings have always
demanded just compensation in exchange for private property, the previous deposit requirement impeded immediate
compensation to the private owner, especially in cases wherein the determination of the final amount of compensation
would prove highly disputed.

Important note:
Read the full case for information on the controversy of setting the actual amount of just compensation.
National Power Corporation v. Lucman Ibrahim
[EMINENT DOMAIN]

Facts:
On November 23, 1994, respondent Lucman G. Ibrahim filed a petition for the recovery of possession of
the land and damages against the NPC. According to Ibrahim, the NPC constructed underground tunnels on the
property of respondents without their knowledge and consent, and without any expropriation proceeding. They
contended that underground tunnels constituted an easement, the building of which should be considered taking under
eminent domain, because it deprived the landowners of substantial enjoyment of their property, and therefore should be
subject to the proper expropriation proceedings and payment of just compensation.

Issues:
1.Is Ibrahim entitled to just compensation?

Ruling:
1. YES. The manner in which the easement was created by the NPC violated the due process rights of
the owners as it was without notice and indemnity to them and did not go through proper expropriation proceedings.
The NPC could have easily secured the valid exercise of the power of eminent domain to acquire the easement over
the property, but it failed to do so. In disregarding this procedure and failing to recognize the owners’ ownership of the
sub-terrain portion, NPC took a risk and exposed itself to greater liability with the passage of time. It must be
emphasized that the acquisition of the easement is not without expense. The underground tunnels imposed limitations
on the owners’ use of the property for an indefinite period and deprived them of its ordinary use. The owners are clearly
entitled to the payment of just compensation. Notwithstanding the fact that NPC only occupied the sub-terrain portion, it
is liable to pay not merely an easement fee but rather the full compensation for the land.
Republic v. Sarabia
[EMINENT DOMAIN]

Facts:
Sometime in 1956, the Air Transportation Office (ATO) took possession and control of some 4,901
square-meter portion of Lot 6068, a 10,468 square-meter lot located at Pook Kalibo, Aklan. Initially, the ATO utilized the
subject occupied portion of Lot 6068 as an airport parking area. Eventually, stores and restaurants were constructed in
the area, wherefore private respondents filed a complaint for the recovery of the possession of land. ATO contended
that the owners of the commercial establishments are in fact its lessees. Petitioner assured private respondents that
they would be paid the fair market value of the subject land. However, the parties did not agree on the amount of
compensation therefor. In 1999, the government filed a petition for expropriation. The RTC fixed the just compensation
for the said lot based on the current market value not at the time of the taking which was in 1956, but at the time of the
issuance of the writ of possession on November 11, 1999.

Issues:
1.Is the RTC correct in fixing the time for which the market value adduced for the determination of just
compensation will be based?

Ruling:
1. NO. The Supreme Court determined that there was sufficient evidence to adduce that the taking of the
occupied lots in fact occurred in 1956. In the context of the States inherent power of eminent domain, there is a taking
when the owner is actually deprived or dispossessed of his property; where there is a practical destruction or a material
impairment of the value of his property; or when he is deprived of the ordinary use thereof. There is a taking in this
sense when the expropriator enters private property not only for a momentary period but for a more permanent
duration, for the purpose of devoting the property to a public use in such a manner as to oust the owner and deprive
him of all beneficial enjoyment thereof. In cases where the taking precedes the petition for eminent domain by a
significant period of time, without the protestation of the landowners, computation of just compensation should be based
on the fair market value during the time of taking, and not the subsequent filing for eminent domain.
Manila International Airport Association v. Rodriguez
[EMINENT DOMAIN]

Facts:
In the 1970s, in line with its expansion project, the MIAA occupied some of the properties surrounding
the area through expropriation. In 1997, Rodriguez bought a lot, a portion of which was occupied by the runway, as well
as all the rights to claim reasonable rents and damages for its occupation, from the previous owner, Buck Estate, Inc. In
a letter dated 20 January 1997, Rodriguez, through counsel, demanded from the MIAA full payment for the property and
back rentals for 27 years, amounting to P468,800,000.00.

Issues:
1.When should the period for the determination of just compensation start?

Ruling:
1. Where actual taking was made without the benefit of expropriation proceedings, and the owner sought
recovery of the possession of the property prior to the filing of expropriation proceedings, the Court has invariably ruled
that it is the value of the property at the time of taking that is controlling for purposes of compensation. Just
compensation is determined on the basis of the value of the property at the time of the taking thereof in expropriation by
the Government, the value of the property as it is when the Government took possession of the land in question, not the
increased value resulting from the passage of time which invariably brings unearned increment to landed properties,
represents the true value to be paid as just compensation for the property taken.
Manila Electric Company v. Pineda
[EMINENT DOMAIN / DUE PROCESS]

Facts:
On October 29, 1974, MERALCO filed a petition for expropriation of the lots of 42 defendants for the
purpose of constructing a 230 KV Transmission line from Barrio Malaya to Tower No. 220 at Pililla, Rizal. Despite
petitioner's offers to pay compensation and attempts to negotiate with the respondents, the parties failed to reach an
agreement. The petitioner strongly maintains that the respondent court's act of determining and ordering the payment of
just compensation to private respondents without formal presentation of evidence by the parties on the reasonable
value of the property constitutes a flagrant violation of petitioner's constitutional right to due process. It stressed that
respondent court ignored the procedure laid down by the law in determining just compensation because it formulated an
opinion of its own as to the value of the land in question without allowing the Board of Commissioners to hold hearings
for the reception of evidence.

Issues:
1. May the court dispense with expert opinion on the determination of just compensation in an
expropriation proceeding and determine the amount for itself?

Ruling:
1. NO. In an expropriation case such as this one where the principal issue is the determination of just
compensation, a trial before the Commissioners is indispensable to allow the parties to present evidence on the issue of
just compensation. The appointment of at least three (3) competent persons as commissioners to ascertain just
compensation for the property sought to be taken is a mandatory requirement in expropriation cases. While it is true
that the findings of commissioners may be disregarded and the court may substitute its own estimate of the value, the
latter may only do so for valid reasons, i.e., where the Commissioners have applied illegal principles to the evidence
submitted to them or where they have disregarded a clear preponderance of evidence, or where the amount allowed is
either grossly inadequate or excessive. Respondent judge's act of determining and ordering the payment of just
compensation without the assistance of a Board of Commissioners is a flagrant violation of petitioner's constitutional
right to due process and is a gross violation of the mandated rule established by the Revised Rules of Court.
Heirs of the Estate of JBL Reyes v. City of Manila
[EMINENT DOMAIN]

Facts:
Jose B. L. Reyes and petitioners Heirs of Edmundo Reyes are the pro-indiviso co-owners in equal
proportion of 11 parcels of land with a total area of 13,940 square meters situated at Sta. Cruz District, Manila. On April
25, 1995, respondent City of Manila filed a complaint for eminent domain on the aforesaid properties, based on
Ordinance No. 7818, which authorized the City Mayor of Manila to expropriate the land to be distributed to the intended
beneficiaries, who were the occupants of the said parcels of land who (had) been occupying the said lands as lessees
or any term thereof for a period of at least 10 years. Petitioners filed a motion to dismiss complaint for eminent domain
for lack of merit, assailing, among other matters, that the City has deprived them of their property without due process
of law.

Issues:
1. Was there a violation of due process in this case?

Ruling:
1. YES. Being a mere creation of the legislature, a local government unit can only exercise powers
granted to it by the legislature. Whether respondent City deprived petitioners of their property without due process of
law depends on whether the City complied with the legal requirements for expropriation. Before respondent City can
exercise its power of eminent domain, the same must be sanctioned and must not violate any law. Section 10 of RA
7279, or the Urban Development and Housing provides that “expropriation shall be resorted to only when other modes
of acquisition have been exhausted.” Private lands rank last in the order of priority (Section 9, RA 7279) for purposes of
socialized housing. In the same vein, expropriation proceedings are to be resorted to only after the other modes of
acquisition have been exhausted. Compliance with these conditions is mandatory because these are the only
safeguards of oftentimes helpless owners of private property against violation of due process when their property is
forcibly taken from them for public use.
Lagcao v. Judge Labra
[EMINENT DOMAIN]

Facts:
Petitioners argue that Ordinance No. 1843, authorizing the mayor of Cebu City to initiate expropriation
proceedings for the acquisition of their lot, is unconstitutional as it sanctions the expropriation of their property for the
purpose of selling it to the squatters, an endeavor contrary to the concept of public use contemplated in the
Constitution. They allege that it will benefit only a handful of people. The ordinance, according to petitioners, was
obviously passed for politicking, the squatters undeniably being a big source of votes.

Issues:
1. Was the aforementioned expropriation by the local government in violation of the Constitution?

Ruling:
1. YES. Local government units have no inherent power of eminent domain and can exercise it only
when expressly authorized by the legislature. By virtue of RA 7160, Congress conferred upon local government units
the power to expropriate. While it is recognized that housing is one of the most serious social problems facing the
country, local government units do not possess unbridled authority to exercise their power of eminent domain in seeking
solutions to this problem. The foundation of the right to exercise eminent domain is genuine necessity and that
necessity must be of public character.Government may not capriciously or arbitrarily choose which private property
should be expropriated. In this case, there was no showing at all why petitioners property was singled out for
expropriation by the city ordinance or what necessity impelled the particular choice or selection. Ordinance No. 1843
stated no reason for the choice of petitioners property as the site of a socialized housing project.
Metropolitan Cebu Water District v. J King and Sons Company, Inc.
[EMINENT DOMAIN]

Facts:
Petitioner wanted to acquire a 5-sq. meter lot occupied by its production well. This lot happens to be
respondent’s property in Banilad, Cebu City. After negotiations have failed, petitioner initiated expropriation proceedings
on November 2004. On February 2005, petitioner filed a motion for the issuance of a writ of possession. Petitioner
wanted to tender the amount to respondent during a rescheduled hearing which petitioners counsel had failed to attend.
Petitioner deposited with the Clerk of Court the amount of P17,500.00 equivalent to one hundred percent (100%) of the
current zonal value of the property which the Bureau of Internal Revenue had pegged at P3,500.00 per square meter.
The Court of Appeals deemed that there was no genuine necessity for the expropriation of the aforementioned lot, and
hat the reliance on Republic Act (R.A.) No. 8974 in fixing the value of the property contravenes the judicial
determination of just compensation.

Issues:
1. Were the expropriation proceedings held in accordance with law?

Ruling:
1. YES. The general rule is that upon filing of the expropriation complaint, the plaintiff has the right to
take or enter into possession of the real property involved if he deposits with the authorized government depositary an
amount equivalent to the assessed value of the property for purposes of taxation. An exception to this procedure is
provided by R.A. No. 8974, which requires the payment of one hundred percent (100%) of the zonal value of the
property to be expropriated to entitle the plaintiff to a writ of possession. R.A. No. 8974 does not take away from the
courts the power to judicially determine the amount of just compensation. The law merely sets the minimum price of the
property as the provisional value. Thus, the amount of just compensation must still be determined by the courts
according to the standards set forth in Section 5 of R.A. No. 8974. Moreover, petitioner was supposed to tender the
provisional payment directly to respondent during a hearing which it had failed to attend. Petitioner, then, deposited the
provisional payment with the court. The trial court did not commit an error in accepting the deposit and in issuing the writ
of possession. The deposit of the provisional amount with the court is equivalent to payment.
Republic v. Lim
[EMINENT DOMAIN]

Facts:
On September 5, 1938, the Republic of the Philippines expropriated lots belonging to respondents for the
purpose of establishing a military reservation for the Philippine Army. After depositing P9,500.00 with the Philippine
National Bank, pursuant to the Order of the CFI dated October 19, 1938, the Republic took possession of the lots.
Thereafter, or on May 14, 1940, the CFI rendered its Decision ordering the Republic to pay the Denzons the sum of
P4,062.10 as just compensation. By 2001, there was still no showing that the respondents were paid just compensation.

Issues:
1. Does the government retain ownership of expropriated lands despite its failure to pay respondents’
predecessors-in-interest just compensation?

Ruling:
1. NO. One of the basic principles enshrined in our Constitution is that no person shall be deprived of his
private property without due process of law; and in expropriation cases, an essential element of due process is that
there must be just compensation whenever private property is taken for public use. The Republic disregarded the
foregoing provision when it failed and refused to pay respondents predecessors-in-interest the just compensation for
Lots 932 and 939. The length of time and the manner with which it evaded payment demonstrate its arbitrary high-
handedness and confiscatory attitude. The final judgment in the expropriation proceedings (Civil Case No. 781) was
entered on April 5, 1948. More than half of a century has passed, yet, to this day, the landowner, now respondent, has
remained empty-handed. Undoubtedly, over 50 years of delayed payment cannot, in any way, be viewed as fair.
Land Bank of the Philippines v. Wycoco
[EMINENT DOMAIN]

Facts:
In line with the Comprehensive Agrarian Reform Program (CARP) of the government, Wycoco voluntarily
offered to sell a 94.1690 hectare unirrigated and untenanted rice land to the Department of Agrarian Reform (DAR) for
P14.9 million. Wycoco rejected the initial proposals for the determination of just compensation, prompting the DAR
administration board to open a trust account in the name of Wycoco and deposit therein the compensation offered while
the property was distributed among farmer-beneficiaries. Wycoco contended that the valuation of the property was
arrived at without factual basis, and therefore justified his rejection of the proposals.

Issues:
1. Did the DARAB err in determining just compensation on mere market value?

Ruling:
1. YES. While market value may be one of the bases of determining just compensation, the same cannot
be arbitrarily arrived at without considering the factors to be appreciated in arriving at the fair market value of the
property e.g., the cost of acquisition, the current value of like properties, its size, shape, location, as well as the tax
declarations thereon. Since these factors were not considered, a remand of the case for determination of just
compensation is necessary.

Important note:
Read the full case for information on the controversy of setting the actual amount of just compensation.
Ivler v. San Pedro
[RIGHTS OF THE ACCUSED – DOUBLE JEOPARDY]

Facts:
Jason Ivler was charged before the Pasig RTC for two offenses, namely: Reckless Imprudence Resulting
in Slight Physical Injuries (Criminal Case No. 82367) and Reckless Imprudence Resulting in Homicide and Damage to
Property (Criminal Case No. 82366). On 7 September 2004, petitioner pleaded guilty to the charge in Criminal Case No.
82367 and was meted out the penalty of public censure. Invoking this conviction, petitioner moved to quash the
Information in Criminal Case No. 82366 for placing him in jeopardy of second punishment for the same offense of
reckless imprudence.

Issues:
1. Will a trial on the second case constitute double jeopardy?
Ruling:
1. YES. The two charges against petitioner, arising from the same facts, were prosecuted under the
same provision of the Revised Penal Code, as amended, namely, Article 365 defining and penalizing quasi-offenses.
The doctrine that reckless imprudence under Article 365 is a single quasi-offense by itself and not merely a means to
commit other crimes such that conviction or acquittal of such quasi-offense bars subsequent prosecution for the same
quasi-offense, regardless of its various resulting acts, undergirded the Supreme Court’s unbroken chain of
jurisprudence on double jeopardy as applied to Article 365. These cases uniformly barred the second prosecutions as
constitutionally impermissible under the Double Jeopardy Clause.

Important note:
This case compelled the SC to ask the National Legislature to re-craft Article 365 so that only the most severe penalty
shall be imposed under a single prosecution of all resulting acts, whether penalized as grave, less grave or light
offenses.

You might also like